ABCD
1
AMA#QuestionAnswer
2
2024.061Brendan Kay says, Suppose you had countless humans in a massive network of interlinked space stations, manually performing digital operations, perhaps with punch cards or some other primitive computational technology, that in total simulate, vastly more slowly, all the sensory inputs and computations of a human nervous system. Would this interplanetary machine feel like a human, or do you think there is something inherent to the size, speed, and or medium of the neuron that allows for our conscious intelligence?
Clearly this is a very similar question to the Chinese room experiment suggested by Frank Jackson years ago, where he said, if we had a room, I'm not going to get the details exactly right, but if we had a vast room with a card catalog inside, where the cards matched up in certain ways, so that you could have input in English, and then, it was, input in English and then, thinking in Chinese or the other way around. I think that there's a person inside who could get input in Chinese, and then go through the card catalog and do various things, various operations, then put out an answer in Chinese that would appear to the outside world like it was conversing. They didn't have large language models at the time, but it's the same basic idea. And Jackson's point was supposed to be, wait, was it Frank Jackson? No, it was John Searle. Oh boy, I'm not on my game today already. It's very early in the AMA. This is John Searle's experiment, of course. Frank Jackson was Mary the color scientist, confusing my arguments for non-physicalism about consciousness. Anyway, Searle's point was supposed to be, surely you don't think that this big room with cards in it is conscious, even though it would pass the Turing test or whatever. And here we're doing something a little bit similar, but with space stations that are interlinked with each other. And the difference is, in Brendan's question, it's not just words coming in and out. You're actually simulating all the neurons exactly. So I don't think that there's anything inherent in the size, speed or medium of actual physical neurons that allows for conscious intelligence. But there are real-world limits to analogies like this also. One is, of course, we are more than our neurons. There are inputs and outputs from those neurons. The neurons in a human being are embedded in a body. They're just cells in the brain other than neurons. And some of those inputs that we get are endogenous, as we say, from inside our bodies. Some are from the outside world or whatever. It might make a huge difference to not have your brain embedded in your body. So maybe you want your system of space stations to include not just neurons, but everything going on in your body and its immediate outside. And having done that, there is an issue of time scales. The time it takes signals to go back and forth from neurons or other cells in your body is relatively short. When it comes to space stations, there's the speed of light, which slows things down. And so the rate of thinking of some kind of chain of space stations like this would be much slower. The being would not have time to have a lot of thoughts in the amount of time it takes human beings to have thoughts. But with all those caveats in mind, sure, that would be 100% conscious. This is what is known as the idea of substrate independence of consciousness. It doesn't matter what the individual pieces are made out of. What matters is the information flow and computation and the results of computation action that goes on as a result of those things. And I get the intuition here because we think of this system of either the Chinese room in Searle's case or the space stations in Brendan's case. We can think of it as living inside, as us living inside it. We're here on Earth looking at all those space stations, bouncing signals around. It doesn't look to us like a living, conscious being. But if you zoom out, so you look at it from afar, so that this set of space stations looks like billions and billions of neurons bouncing back and forth, and you slow down your view of it so that it is effectively thinking just as fast as a human being, it would be just like a human being. It would be just as conscious as that. So I think that rather than actually being an argument against the physicalism of consciousness, these kinds of arguments are arguments against taking your intuition too seriously when you push that intuition into circumstances that are far, far, far beyond actual experience that we have in our everyday lives. 'Cause our experience is trained on certain subsets of possible things happening, and therefore when you imagine things outside that subset, it's easy to go astray if it's your intuition that you're relying on.
3
2024.062Matthew Hines asks a priority question. Remember that priority questions are for patrons, people who support on Patreon, can not only ask AMA questions, but unfortunately we don't get a chance to answer all the AMA questions 'cause there's too many of them. So the priority question idea is that once in your life, you get to ask a question and I will do my best to answer it. So Matthew's question is, I'm working on a science fiction story, I'm wondering if you could help me with something. For reasons that are vital to this story, I need a method of space travel that circumvents the light barrier, which is not a warp drive. So in other words, he needs something that is not a warp drive, but nevertheless lets you go faster than light. My conception is a burst of energy that tears a hole in the space-time fabric, giving access to a kind of hyperspace through which vessels can pass. A trip to Mars from Earth is done in a subjective instant, while trips to nearby star systems take a few minutes to a few hours, with the further reaches of our galaxy taking no more than a few days. Can you think of anything that would give scientific gloss to this idea? I realize it's a horrible affront to the laws of physics, but please trust me when I say that it's vital to the story and it absolutely cannot be warp drive.
I mean, I can tell you the obvious answer, which is you need a temporary wormhole. Wormholes are what you need when you don't wanna use a warp drive and you wanna get across vast distances in a short period of time. You need some shortcut through space-time. And having a wormhole, a little tube that connects two different regions of space-time with a much shorter distance than it would take if you didn't go through that tube, it's very natural for wormholes to be temporary. They want to be temporary. In the real world, as far as we know, even if you could make a wormhole, you couldn't actually travel through it because it would just collapse quicker than you could possibly travel through. There's always the limit of the speed of light travel when you're in the wormhole, and the wormhole tends to collapse faster than the time it takes light to go from one end of the wormhole to the other. So wormholes are generically non-traversable. Now that's not a theorem. You can always play with things and try your best to build traversable wormholes, and that's what Kip Thorne and his friends talked about many years ago. But it's not at all hard to imagine that if you allow yourself the gimmick of making a wormhole that you can travel through, then it would effectively close very quickly. So it wouldn't be like a permanent subway tunnel. It's just a temporary shortcut. One thing to keep in mind is when you say things like, trips to nearby star systems take a few minutes to a few hours, you still have got to remember the theory of relativity. What does it mean to take a few minutes to a few hours for something that is effectively faster than the speed of light? You have to take relativity into account. In whose reference frame is it taking a few minutes to a few hours? That's potentially an answerable question. Maybe it's in the reference frame defined by the people who first did the explosion or whatever. I'm just saying that you have to take that into consideration.
4
2024.063Anonymous says, as an expert in the field, what would you suggest is my responsibility to raise a voice that my field is getting excessive funding relative to a realistic expectation of societal benefit? I'm working in quantum computing adjacent fields for about 10 years, and I think you could classify me as a specialist. Based on my understanding and extrapolation of the rate of progress, I feel confident that there's no reason to expect it to provide practical benefits in the foreseeable future. It is pretty clear that the investments in the field are based on false premises, plus you have to oversell to get the money attitude. The investments are also at the level of spending by governments for major humanitarian causes. So sadly, I feel obliged to advocate for reducing the funding. As you can imagine, that's very unpopular. What would you say I can or I should do?
I would say, this is an individual thing. I think you have to answer for yourself what your standards are. But I think roughly, there's not a moral argument that says you have to advocate for less funding for a field you think is interesting. I mean, if you think it's intellectually interesting, apart from the possible benefit to humanity, then you can make a good faith argument that it's worth funding. What I do for a living, I do not do because of potential improvements in human standards of living or anything like that. I do it because we wanna know the answer, and people are willing to pay some money to know the answer. We paid $10 billion for the Large Hadron Collider with essentially zero promise that it was going to lead to any technological benefit. We just want to know the answer. You can say the same thing for space telescopes and things like that. On the other hand, you shouldn't lie. You shouldn't say that there will be improvements in technology or whatever if you think that that is not true. I think that in the fields that I am in anyway, I feel perfectly happy saying true things about the value of my field and advocating for funding for it. There is nothing wrong with that. The issue here is not whether or not your field will lead to improvements. That is hard to say. Maybe you are right that it won't or is unlikely to, but it is hard to say. But it is easy to say that you shouldn't lie, that you should do your best to tell the truth about what the prospects are and nevertheless make whatever arguments you should or you can think of for funding for the field.
5
2024.064I am going to group three questions together. This was a common topic this week and for very good reasons. A goon in the gooniverse says, when a physics student starts out they learn about particles. In grad school they find out particles are actually excitations in fields and fields are fundamental things. Feynman gave us a way to understand field interactions using particle-like pictures, but it's all field theory. Similarly, when a student opens a textbook on string theory, it will begin by minimizing the action of a string's worldsheet. But what are the underlying objects in string theory? Are strings like more complicated vibrations in a field, or is it something else? Chris Mason says, I have listened to Mindscape for a number of years and watched all the biggest ideas in the universe video series. Looking back, I am delighted with how much I have understood, including a popular level understanding of quantum field theory. However, listening to your latest podcast on string theory, I realized it has not clicked for me where string theory fits in with quantum field theory. If string theory turns out to be correct, does that mean QFT is wrong, or would it be better to think of QFT as a limit of string theory in a particular regime? And finally, Connor says, most succinctly, which I am in favor of, everyone who can write their questions succinctly, please try to do that. Quantum field theory says particles are really vibrations in quantum fields. String theory says particles are really strings. What are strings from the perspective of quantum field theory?
These are perfectly reasonable set of questions, ones that I had had myself back in the day, especially when I was a grad student first hearing about these ideas. You learn when you take quantum field theory that particles, as we've said here, are excitations in quantum fields. A quantum field, When you take a field, which doesn't look at all particle-like at the classical level, and you apply the rules of quantum mechanics to it, so just like a harmonic oscillator or an electron in an atom, suddenly you have energy levels that are discrete. And it turns out that those energy levels behave and act like sets of particles. So a quantum field can simultaneously describe zero particles, one particle, two particle states, and so on, all the way up to as many particles as you might want. So in a very real sense, that's a unification of the idea of particles and the idea of fields. It doesn't mean you can't talk about particles. Particles are perfectly good ways of talking about it. That's what Feynman diagrams do. But it suggests that at a philosophical level, the fields are the more fundamental things. And then string theory comes along and string theory tries to generalize the idea of particles, which are point-like things, zero-dimensional, to the idea of strings, which are little one-dimensional, either line segments or circles. And you can talk about those strings interacting and vibrating in different ways, etc. So it is very natural to ask, what is the string theory version of field theory? What is the more fundamental thing, the excitations of which look like vibrating strings? And people have tried, people talked about doing this. There is something called, guess what, string field theory. And you can look it up. There is actually just a review article by Ashoke Sen and Barton Zwiebach just the other day that appeared. So it's an ongoing research area, what is string field theory? It is slowed down for a couple of reasons. One is we are not really sure what string theory is. We are pretty sure what quantum field theory is and what particles are. But you can see papers by people like Joe Polchinski called What is String Theory? And we are not exactly sure what the answer is. And you might say, well, how do we not know what string theory is with all these people working on string theory? The answer is we know certain aspects of string theory. We know certain versions of it and certain limits as we talked about with Cumrun Vafa, etc. Without necessarily knowing the complete once and for all story of it. That's not actually so difficult to wrap your brain around. So it's not clear whether it is useful in the context of string theory to do the same thing you did with quantum field theory. That is to say to come up with a string field, a string version of field theory, the excitations of which are strings. Now, string theory reproduces conventional quantum field theory in the limit where the length scales you're looking at are much larger than the sizes of the string. And usually the strings are the Planck scale across. So as long as you are looking at lower energies than that, longer wavelengths than that, things will look like quantum field theory. Then, string theory reproduces the successes of quantum field theory. But whether string field theory is the best way of thinking about string theory still seems as far as I can tell to be an open question. One more important thing to say here is there's different ways of thinking about quantum field theory. There's one way which says start with a classical field, quantize it, see the particles pop out. That is the way that I like to think about it myself. That's the way I talk about it in the podcast we did recently, the solo podcast or in the book, Quanta and Fields. Start with the field, quantize it, particles come out. But there is another way of thinking about quantum field theory which starts with the particles. Start with a quantum mechanical theory of different kinds of states, different factors of Hilbert space, one of which describes zero particle states, the vacuum, one of which another part of Hilbert space describes one particle states, another part describes two particle states and so forth, the whole collection of all of them. And invent the rules that obey features like locality and Lorentz invariance and things like that for interactions of the particles within this theory that allows for different numbers of particles to be created or destroyed. And what you discover by doing that is that you have reinvented quantum field theory. So in other words, at least in certain limits, let's put it that way, at least in certain circumstances, the map between fields and particles goes both ways. You can either start with a classical field quantize and get particles out, or you can start with an arbitrary and possibly changing number of particles and superpositions of number particles and discover that it's really quantum field theory. So when people do string field theory, it's more like that ladder way of doing it. It's not, again, I'm not an expert on this, maybe things have changed, but in my very limited experience, they don't start with something that is the string theory version of field theory, quantize it and find strings popping out. They start with Hilbert space that can describe different numbers of strings and allowing those strings to interact and change the number of strings and so forth and put it all together to make something called a string field theory. Whether or not that that has been very successful so far, you would have to read the papers out there, but it can be done.
6
2024.065Mark Slyte says, could you talk a bit about this problem of energy not being conserved when a wave function collapses? How problematic is it? Is it treated differently in different quantum mechanical theories? Is it a serious problem?
Yeah, it's not a problem at all. It's zero problem, really, it's just a fact. For those of you who don't know, well, in quantum mechanics, there are two different ways that wave functions evolve according to the conventional Copenhagen textbook version that we tell our students about. One way is the Schrodinger equation. That's how wave functions evolve when we are not measuring the quantum mechanical system. And when that's true, as long as the laws of physics, the Schrodinger equation, at the technical level, the Hamiltonian, as long as those are independent of time, then energy is perfectly conserved. And that's exactly like classical physics. As long as the laws of physics are independent of time, Noether's theorem tells us there's a conserved quantity, that's energy. But in quantum mechanics, we have this other way for quantum states to evolve, and that is what happens when you do a measurement. The wave function collapses, you're in a different state after the measurement. You're in the eigenstate of whatever observable it is that you are measuring. And typically, you have two options here. One is you can say, well, there's no such thing as energy. I don't know what energy is, so it's meaningless to say that energy is not conserved during that process. Some people take that attitude, okay. But there's another thing you can do, which is a more straightforward thing, which is to say, look, when you weren't measuring it, there is a conserved quantity that is an energy. It is what we call the expectation value of the Hamiltonian, the expected value of the energy under observations. And so if you have a state where you can observe different things, and maybe you can observe the energy directly, and you get an outcome, but you're in a state where you could get different outcomes for the energy, just like if you're in a superposition of different spins or positions, you could get different outcomes for that. You can take the average. You can take the weighted average of the value of energy you might observe times the probability of observing that. And that quantity, the expectation value of the energy, is perfectly 100% conserved under the Schrodinger equation. That's exactly the quantum mechanical version of Noether's theorem. It is not conserved when wave functions collapse. And this is one of those facts that is sort of both trivially true and very surprising to people, even professional physicists, when you tell them about it because they will say, well, that's because the energy went into the environment or your measuring apparatus or whatever. And no, that is not right. They are wrong. So Jackie Laudman and I wrote a paper very explicitly pointing this out. As usual, as predictable, the response to the paper was half the people said, this is so obvious, why are you even bothering? And the other half said, this is completely wrong, why are you even bothering? So that's why it was a worthwhile paper to write, I think. But it's not a problem. Why would it be a problem? It would be a problem if it didn't fit the data. That's what's a problem. You might not like it. You might think it is inelegant. Or maybe you want to hope that the ultimate laws of physics don't have this feature. That's okay, because we don't know what the ultimate laws of physics are. As we point out in our paper, whether or not energy really is conserved will depend on your favorite attitude toward the foundations of quantum mechanics. In many worlds, it is conserved in the wave function of the universe, because the wave function of the universe always obeys the Schrodinger equation. It is not conserved from the point of view of any one observer, because one observer does not see the whole wave function, they only see the branch that they're in. And so experimentally, you could absolutely see the average energy change. It's hard to know that it's changed, 'cause it's hard to know what it was before and after, but in principle, you could see that happen. In other theories, like objective collapse theories, the energy just changes, just not conserved during wave function collapse. Up until you do an experiment that either confirms or falsifies this prediction, there's nothing problematic about it, it's just a prediction.
7
2024.066JMS547 says, in your solo episode outlining your new book, you mentioned at one hour and seven minutes that you originally wrote a lot more about symmetry and group theory. Will you ever be releasing that extra material?
No, I don't think I will. If you ever watch versions of movies that include, after the movie you get deleted scenes on the DVD or whatever, almost always there are good reasons why those scenes were deleted. They're not as good as what's in there. And sometimes this happens when you're writing a book. Not only do you write more than you need, but some of it you realize, yeah, you didn't really need to write that, it's not very good. So the parts that I wrote about symmetries and whatever are dug into details about discrete like the triangle group and the square group and whatever, and a little bit of details about what kinds of matrices represent different smooth group transformations like SU2 and SU3 and so forth. All of it is sort of small, it's specific and not that deep really. So I don't think the world really needs that. There's better places to go if you wanna look about into group theory for its own sake rather than as we use it in physics.
8
2024.067Tyler Smucker says, in one of the biggest ideas in the universe videos, you were wearing a shirt that said, licensed quantum mechanic. Where did you get that shirt? I want to buy one.
You can't buy it. Sorry. Those shirts are given out to people who either have taken and passed or taught physics 125C at Caltech. At Caltech, there is a year-long course for juniors on quantum mechanics physics 125 A, B, and C, because we're on the quarter system at Caltech. And Mark Wise, who taught 125C for a long time, invented a tradition where at the end of the day, at the end of the quarter, he would hand out free t-shirts to everyone who passed the course. And so I taught the course one year. I followed up the tradition, and I got my own t-shirt that year, licensed quantum mechanic. So I can't give it to you unless you went to Caltech and passed the course. However, I will point out, as long as you promise not to tell anybody, it's not that difficult to get a black t-shirt made with whatever words on it you want. So go for it.
9
2024.068Nikola Ivanov says, is each point of the vacuum state a virtual superposition of all the quantum fields of quantum field theory and random quantum fluctuations? Can potentially make this point to be part of a quantum state of any field? In other words, is each point of the Hilbert space pregnant with all the particle gauge and scalar fields of the theory?
I'm not sure I understand this question, sorry about that, but let's approach it slowly. Let's think classically first, okay? Classically, at every point in space in a field theory, in a classical field theory, there will be different kinds of fields. Maybe the electric field, and the magnetic field, and the gravitational field, and the Higgs field, or whatever. Those are all perfectly good classical behaving fields. It's easier to imagine classical behavior for bosonic fields than fermionic fields, but you could do it for fermions also. But the point is that they're all separately there at every point. So at the classical level, the space of states involves what each field is doing at every specific point. Now once you're in quantum field theory, you create a Hilbert space out of superpositions of all those possibilities. So I think maybe this is what you're getting at. The Hilbert space is the set of all possible superpositions of all possible values of all the fields at every point. There's a lot going on in quantum field theory, but yes, the fields are there at every point in possible superpositions of everything you could possibly measure about them.
10
2024.069Eric Chen says, love the new episode with Cumrun Vafa. Could you give an intuitive explanation for what compactification is? What exactly does it mean that extra dimensions are compactified onto a six-dimensional torus or a Calabi-Yau manifold? Possibly, probably nonsensical third part of the question. Could we have classical theories in say 11 dimensions, but with the extra dimensions suitably compactified so as to yield our standard classical macroscopic physics?
So yes, even let's go with the third part first. Classically, absolutely. You could imagine classical compactified theories. Indeed, the whole idea of compactifying, which was really first pursued by Kaluza and Klein, was an immediate spin-off of general relativity before we had a complete theory of quantum mechanics, much less quantum field theory. Kaluza and Klein noticed that once you had general relativity, so once you said that space-time itself has curvature and a metric and is dynamical, you can imagine that a dimension of space-time is compact. Compact doesn't mean small. It means in strictly mathematical language, it means topologically it's compact, which means it's not infinitely big, but it could still be finite, but big. So a compact dimension could be topologically a circle, but it could be a very, very big circle. Indeed, it's possible that all three of our observed dimensions are compact in some sense. However, usually, informally, when we say compactified as physicists, we also mean small, invisibly small to us. So Kaluza and Klein realized that if you had a circle as an extra dimension, there's instantly symmetry in your theory because you can put whatever coordinates you like on that circle. So this is a gauge theory. This is a gauge invariance because at every different point in space, you could separately rotate your circle. And indeed, that is a u1 gauge invariance. If you put coordinates so that the circle is co-ordinatized by complex numbers, like e to the I theta, where theta goes from zero to two pi, and theta tells you where you are on the circle, I can separately rotate theta at every point in space. That is a u1 gauge invariance, almost exactly the same as electromagnetism. This was the reason for the original excitement about Kaluza-Klein theory back in those days. Maybe it is a way to unify gravity electromagnetism. Einstein was extremely excited by this possibility. It doesn't give you exactly conventional electromagnetism because you have another thing that you can do with that circle, which is it can get bigger and smaller. So To the outside world, to people who are much larger than the size of the circle, that is a field. The size of the circle shows up as a field, as a scalar field in our ordinary macroscopic world, sometimes called the dilaton or the Kaluza-Klein scalar or what have you. And We don't have any evidence for such a field in the real world. And Furthermore, you would like it so that you not only compactified the extra dimension or multiple dimensions, but you stabilized them as well. And This is something you can talk about classically. The size of the extra dimension and the shape for that matter doesn't want to shrink down to zero or expand to infinity. When Cumrun Vafa was talking about the difficulty of getting a positive cosmological constant in string theory, one of the things that is driving that intuition is that when you compactify extra dimensions, there's always a way to make the vacuum energy zero by making the extra dimensions bigger. And so if the extra, if the vacuum energy is positive and there's another way that you can arrange extra dimensions of the vacuum energy is zero, that's going to have lower energy. Zero is a lower number than any positive number. So you can't perfectly stabilize the extra dimensions, but maybe you can temporarily stabilize them. That would be the hope. So anyway, that question is the easy part of your question to answer. So I'm not quite sure what you mean by an intuitive explanation for what compactification is. Let me give it the following stab. Remember that we're doing field theory. So remember that we, 'cause as I said before, string theory reduces to field theory in the limit where you're much bigger than the strings. Let's imagine we're doing that limit, okay? So we're doing field theory on space-time multiplied by a small compact spatial manifold, like a circle or some six-dimensional globular manifold or whatever. So you have fields that can take on every value at every point in our observable space and in this little compact space, okay? So the fields take on values everywhere, but the compact spaces are very, very small. So if the fields are exactly constant over the extra dimensions, that's fine. That doesn't cost any energy. It costs energy when the fields change from place to place, but when they're constant, it doesn't cost any energy. And if the extra dimensions are very, very small, then if those fields change at all from place to place within the extra dimensions, it costs a lot of energy. So therefore, in our low energy late universe where we live right now, it's almost like the extra dimensions aren't there. They give rise to extra fields like that dilaton field or maybe like the electromagnetic field or other gauge fields, etc. But they don't, you can't see them. You can't see them directly because they're small. It takes too much energy to see them. In quantum field theory, short distances are coupled to high energies. And so that's why people will sometimes say you need to build a particle accelerator that smashes particles together at the Planck energy in order to reveal the extra dimensions or other signatures of string theory. So of course you're gonna get different signatures depending on what the extra dimensional shape is, but that's why we pay string theorists to figure that one out.
11
2024.0610Emmett Francis says, I just had the chance to teach my first lecture in an engineering mathematics course and it was challenging. Do you have any go-to tips for keeping students engaged during lecture while effectively and efficiently teaching the material?
I mean I don't have a short list or anything like that. You know I think it depends a lot on what you're teaching, what the goals are of the course. I do vividly remember one anecdote that was in the introduction to a book, a textbook, that I used as an undergraduate for a course on mathematical physics. So I don't know, you say engineering mathematics, I'm not exactly sure what that involves, but very often in an undergraduate physics curriculum there will be a course called mathematical methods for physicists and you'll learn about Fourier transforms and complex analysis and other things, you know vector calculus, things that appear over and over again in different physics courses, better just to teach them in a single unified math course so the thinking goes, than to sprinkle all of those nuggets of wisdom among many different actual physics courses. Whereas something like differential geometry only appears in your general relativity course and therefore you learn it there, okay. But anyway, this intro to the textbook said, the author in his voice was writing, when I first started teaching, I forget what the book was, sorry so I can't give the person credit, when I first started teaching mathematical physics, the students kept asking me for applications of the different ideas. And so I went to one of my senior colleagues and said, what do you do when your students ask you for applications of the concepts you're teaching in mathematical physics? And the senior colleague said, I give them to them. The idea being that if you're teaching an engineering math course or a math methods for physics or for biology or whatever, presumably every mathematical thing you're teaching is being taught for some reason, because you use it for some engineering purpose down the line. So let them know that. Don't insist that we're gonna study math for its own sake. Studying math for its own sake is Great And there's a way to do that, namely take a math course and they can prove theorems and have epsilons and deltas and what have you. But for a kind of applied math course, it's super important to keep people aware of why you're learning this. I tell the slightly amusing story in Quanta and Fields that when I took that very mathematical physics course I'm talking about, when we were first taught Fourier transforms, I thought, wow, this is just totally useless and tedious, but I guess I got to learn it just to pass the class, even if I never use it again. Of course, it's the most useful thing in the world once you start doing field theory and quantum mechanics. So tell people why you're doing it. I think that's the single most important thing for the question you ask, which is how to keep people engaged. Of course, how to be a good teacher, how to get the lessons and the ideas across, that involves a lot of work, a lot of different strategies for that, probably more than I have time to go into right now.
12
2024.0611Razi Ahmed said, Cumrun Vafa made a comment that the dimensionality of space at the very beginning is ambiguous, I.e. It's not clear whether to think of space as a zero-dimensional object or just a very small three-dimensional object. Did I understand that correctly? If it is that space was a zero-dimensional object at the beginning, did space ever go through a one or a two-dimensional existence?
Well, we don't know. I think the right way to interpret what Cumrun was saying was when you get either the compact dimensions or all the dimensions that we live in right now small enough, small enough that quantum gravity becomes important. You know, just as we say over and over again in quantum mechanics, there's not really any such thing as the position of the electron. There's a wave function that can tell you the probability of seeing different outcomes if you measure the position of the electron, but there is not a pre-existing thing called the actual position of the electron. Likewise, for dynamical space in quantum gravity, there's no such thing as the size of the extra dimensions or maybe even its dimensionality. That's sort of a limiting concept in the big classical limit. We don't know what was going on near the Big Bang as far as the dimensions of space-time are concerned. Cumrun Vafa actually wrote a well-known paper with Robert Brandenberger may years ago where they studied a scenario where all... So if you're a string theorist and you believe there are 10 dimensions of space-time, therefore 9 dimensions of space, they imagined a scenario where all 9 dimensions were compact and small and they pointed out that if you had strings wrapped around these dimensions, you can easily come up with a scenario where you make use of the fact that the thing that is special about three dimensions of space is that strings intersect in them. If you have strings moving in three-dimensional space, they will generically run into each other. If you have points, two dots in three-dimensional space moving around, they will generally never hit each other. You can infinitely finely tune them so they do, but generically they won't. Whereas strings in three dimensions will generically hit each other, in four or more, even strings will not generically ever intersect each other. So, Cumrun and Robert invented this theory where three dimensions of space started growing because basically the strings that were holding them together unwound because they kept intersecting in three-dimensional space. So that's the Brandenburger-Vafa cosmology scenario. I think it's cheating a little bit. I think that it doesn't really work for arrow of time type purposes, but it's certainly a very fascinating idea. I often had the opposite idea. So like motivated by the arrow of time, I've always thought that these initial conditions that cosmologists tend to imagine are super-duper finely tuned and unfair. So why would all the dimensions of space be small at large times? So I've often imagined, well, what if all the dimensions were large? Could you compactify some of the dimensions? Could you go from all large dimensions to smaller dimensions? And I wrote a paper with Matthew Johnson and Lisa Randall where we suggested exactly a way to do that. Our method doesn't quite play well with string theory 'cause it's based on positive cosmological constants, but it is an intriguing possibility. So the point of this, the lesson here, is that we have no idea whether how many dimensions were big or small at very very early times. This is something where we have no data so we're free to speculate.
13
2024.0612Vicken Vorperian says, why is Petrus wine one of the most expensive Bordeaux wines? About $2,600 for a regular-sized bottle. Did you get to taste it when in France, and is it worth it?
So no, I have never got a chance to taste Château Pétrus. I have gotten chances to taste various expensive Bordeaux wines. Probably the most expensive I've ever tasted was a bottle of 1982 Château Margaux. For these expensive wines, the vintage, the year that they're from, makes a big difference to the quality and the price also. '82 was apparently, I'm not quite enough of an expert to really verify this from experience, but it is said to be a classic, amazing vintage for Bordeauxs. And so Château Margaux is one of the first growth, that is to say, highest quality by the old classification scheme, Bordeaux wines. And I didn't buy the bottle. I think Château Margaux '82 would probably be around $1000 right now. So we didn't buy it, it was served at a dinner that I happened to be lucky enough to participate in. Yeah, it's better than what you get if you pay $100 for a bottle of wine or $20 for a bottle of wine. There's a lot of variation here. It's not a strict relationship between cost and quality, especially because different people will experience the wines differently. And this is just science. Different people have different taste receptors and different experiences, different sensitivities. So if you personally, not necessarily you, Vikram, but if one personally has their favorite wine in the world that is $20, then they should absolutely go for that. There's no objective sense in which the more expensive ones are better. But there is an objective sense in which the more expensive ones are different. They last longer, they're more complex, they have more structure, they have more subtlety. An '82 in a cheap bottle of wine is not going to be very drinkable right now. It's well past its prime, but these classic great bottles of wine can last 50 years and still be pretty awesome. Is it worth it? That depends on your income level. Was it worth it when I was a graduate student to drink a $2600 bottle of wine? No, because I didn't have enough money to do that. Is it worth it now? Also no, probably not. I think that if I splurge or whatever, I could imagine at this point in my life spending a few hundred dollars for a bottle of wine if it's a special anniversary or something like that. But a couple thousand dollars is still beyond my reach. But if someone just gifted me $10 million and said, "You're not allowed to give this away to charity, you have to spend it on yourself," would I buy a few bottles of Château Pétrus and see what the fuss was about? Sure, I'd be very happy to do that. There are bottles of wine that are $40,000 or something like that. There are silly bottles of wine. Also, when I was in France, we took a little class about Bordeaux wines and the guy mentioned, I think it might be Château Pétrus. No, maybe it was Cheval Blanc, I forget. The most expensive bottle of wine ever sold. But it's not because it was the best bottle of wine ever sold, it was because a few bottles were brought up to the International Space Station and then brought down again. It's just like these bottles from Thomas Jefferson's time or whatever. Many of which were fake, but you're buying it for the gimmick. You're buying it for collector value, not because it's the world's best wine. There are $40,000 bottles where the claim is that that's worth it, that is worth the improvement in quality if you can afford that price. I have no experience at that verified level, so I can't tell you whether that's just whistling in the dark or something to talk about. But anyway, yes, if I had infinite amounts of money, I'd be very interested in trying it. I completely disbelieve people who say there's no difference between a $1000 bottle of wine and a $20,000 bottle... And a $20 bottle of wine. I think those people are just sour grapes. So there clearly are differences. Again, you don't have to like those differences, but they're there.
14
2024.0613Timothy Altman says, "I believe you would agree we have little control over adoption of technology on a global scale. AI, designer babies, and fossil fuels are all examples. Yet so far, we've been smart enough or lucky enough to mitigate downsides. Are you optimistic about the future of the human race or do you think we are just rolling the dice over and over again until they come up snake eyes?"
That's quite a journey through your question there, Timothy. The one thing to keep in mind is that the past is not necessarily a good guide to the future because as technologies develop, the leverage that these technologies have is much greater. 200 years ago, the most powerful technologies we have just didn't have the capacity to wreak damage on the world in the same way that modern technologies or future technologies do. So we can't just make an argument that says, "We figured it out before, it'll be okay again." I think it's legitimate to consider the challenges that we have now to be of a different order than the ones we had before. Secondly, the fact that we've survived so far, there's a bit of a selection effect there, isn't there? You survive up until you die. And so it's very hard to extrapolate from your past successes, because if you hadn't succeeded, you wouldn't be here to talk about them. In terms of mitigating downsides, I don't know. We've mitigated some of the downsides. We're also causing disastrous changes to our climate, so we haven't done it perfectly well. Many, many people died from pollution and such things like that in the Industrial Revolution. Many people died when we dropped atomic weapons on Japan. There's lots of bad sides that we did not successfully mitigate. So I think it's a mistake to think that we know now whether we should be optimistic about the future of the human race or not. There's plenty to be optimistic about. There's plenty to be pessimistic about. We have to take an active role in trying to increase the chances for the optimistic scenarios and decreasing that for the pessimistic ones.
15
2024.0614Richard Cashton says, "I don't know if this question even makes sense. Consider time as the fourth dimension. I want to make a four dimensional cube, I.e. The same size in all dimensions. I build a cube very quickly, wait some amount of time, and then destroy the cube quickly. This will create a 4D shape. But how long should the time be in order to equal the length of the cube? I'm guessing the amount of time it takes for light to travel that distance."
Yes, that's exactly right. Or at least that's the closest you have to a meaningful comparison. Space and time are different. They're both part of space-time, but we measure them using clocks versus rods. Of course, there's a conversion factor between them, and that conversion factor is exactly the speed of light. So roughly speaking, what you wanna do to do what you're asking is measure distances in light years and time in years, or distances in light seconds and time in seconds, which means basically that for any human scale distance measure for the cube that you're building, the time scale is going to be incredibly, incredibly tiny so that all the lengths of the 4D cube are equal or at least commensurable in some sense.
16
2024.0615Christian Hoffman says, "In your recent book, Quantiton Fields, you mentioned that the wave functions of particles spread out as time passes and that larger objects spread out slower. Why is that the case and how fast do everyday objects actually spread out?"
Well, how fast actual everyday objects spread out depends on details, right? The details depend on the details, unsurprisingly enough. I think that if you wanna sort of look past the details to get a moral of the story, why is there some relationship between the mass of things and how quickly their wave functions spread? The answer is basically because the natural variables to discuss quantum mechanical systems are position and momentum. So in classical mechanics, we would say, I could completely determine the state of a system by giving you the position and velocity of every component. Or equally well, I could give you position and momentum because momentum and velocity are related. Momentum is mass times velocity, right? But it turns out in quantum mechanics, and even in classical mechanics, if you look at it carefully enough, once you start talking about phase space in classical mechanics or conjugate observables in quantum mechanics, as appear in the Heisenberg uncertainty principle, the natural thing to do is to use momentum, not position. So the uncertainty principle is not the uncertainty in position times the uncertainty in velocity is greater than h-bar, the natural quantum unit. It's the uncertainty in position times the uncertainty in momentum is greater than h-bar. So if you have a certain momentum of something and you kind of fix that, then for different values of the mass that corresponds to different velocities. And in particular, when the mass goes up, when you get something heavier and heavier, you need a smaller and smaller velocity to get that same momentum. So if you have an uncertainty in the velocity, a little uncertainty in the momentum, a little spread in the momentum part of the wave function, as you might say, for heavier objects, the actual speed, the velocity at which the spread happens will be much slower than the speed with which the spread will happen for a low mass particle, just because momentum is mass times velocity. I think that's the essence of what you're asking about here.
17
2024.0616Dragon-sided D says, "What are your thoughts in the current situation with Gaza, Hamas, and Israel?"
Yeah, well that's a complicated one. I think it's tragic. That's the simple-minded thought about it. I think that there is a lot of blame and badness to go on all sides. I also think that not all sides are created equal. But they're all bad. They're all doing bad things in that sense. I think that neither side, Palestine or Israel, has leadership which fairly reflects the views of the people on either side and I think it's a shame that the general people get blamed for some of the things that the leaderships do on either side. I also think that Israel is a much more powerful entity than Palestine is and that creates a huge imbalance. Israel can do a lot more damage. The number of deaths caused by Israel in the the conflict between Palestinians and the Israelis is much larger than the number of deaths caused by the Palestinians. That's not to excuse the terrible terrorist attacks that Palestinian terrorists have pulled off. It's all bad. So it's just all bad. I think it's terrible. I would like a ceasefire right away. Ultimately, I would absolutely like the Palestinian people to have a nation of their own and they could govern it on their own. And I think that if Israel were smart, rather than trying to starve people and bomb infrastructure and destroy hospitals and universities, they would flood the Palestine with money. They would support the wellbeing of Palestinians to lead comfortable, happy lives in their nation, and then it would be much less likely and less tempting to have terrorist attacks against Israel. But no one seems to be listening to me when it comes to those particular geopolitical issues.
18
2024.0617Tim Giannitsos says, "David Deutsch argued during your conversation that he was not a Bayesian but he is an Everettian. Is this a contradiction? It seems that an Everettian must speak a probability in terms of weightings or credences like a Bayesian would as opposed to the limiting behavior of many identical experiments like a frequentist would."
Well, I probably shouldn't have answered this question honestly because I do have a policy that I don't wanna try to interpret what other people are saying if I don't understand what they're saying very well. I have not understood why David Deutsch doesn't like Bayesianism, even though we talked about it on the podcast. I still came out of it not quite understanding that. I know that part of it is a philosophy of science question. Of course, by the way, Bayes has a theorem, right? There's Bayes' theorem, which relates the probability... Certain conditional probabilities to other conditional probabilities and can be used to update your credences when new information comes in. Nobody disagrees with the theorem, not David Deutsch or anybody else. That's not what it means to be not Bayesian. There is something that goes beyond the theorem, which is to say Bayesian epistemology, to say that the way we should think about hypothetical scenarios is to assign them all credences and then update them when new evidence comes in. And David thinks that that doesn't work. He thinks it doesn't actually do what you want it to do. He has a paper that he has coming out, or maybe it came out already. We did talk about it during the podcast that basically argues following a theorem of Karl Popper that the kinds of things you learn about when you update in using Bayes' theorem are only more or less redundant with whatever evidence you collected. They're not actually telling you anything about independent parts of the theoretical structure you're considering. Again, I don't quite see why that theorem gets him where he wants to go, but he's a Popperian when it comes to the philosophy of science. Popper, remember, emphasized conjectures and refutations. That was the name of one of his books, and the idea of falsifiability, the idea that the way that science goes forward is you put every possible conjecture out there that you can think of, and then you falsify. You refute as many of them as you can, and what is left standing is supposed to be closer and closer to the truth. Notice that the word credence never appeared there. So there's a slight change in emphasis. But that's about theories, not about which world you live in, okay? So maybe Dave would be perfectly happy to be a Bayesian about what branch of the wave function you end up in. Again, it's not what he actually does. He actually when he first very famously offered up a derivation of The Born Rule in many worlds, he did not use Bayesian language. He did not use credences or anything like that. He just used expectation values and decision theory and stuff like that. So it's possible he would argue that even in that choice, not what theory do you choose, but what branch of the wave function are you on, there's no good reason to be Bayesian about things. Again, I probably shouldn't say, you should probably ask him.
19
2024.0618Kilmgod says, "Why is a photon its own antiparticle as you say in quantum fields?"
It's not exactly. I forget exactly what I said. It's a little bit sloppy to say that a photon is its own antiparticle. The more honest thing to say is that not all particles have antiparticles. The idea of an antiparticle... Let's put it this way. There's a whole bunch of fields that exist and various features of quantum field theory and the mathematics underlying what fields you're dealing with indicate that you need to have certain properties and certain symmetries within those fields. And when your fields carry a conserved quantity, like spin or electric charge or things like that, quantities that can take on opposite values, spin up, spin down, charge plus one, charge minus one, then you will have something like an antiparticle. Actually spin is probably a bad example, isn't it? Photons have spin and they have equal and opposite spins in two different helicities, as we say. There's a positive helicity, which is spin clockwise along the line of motion and there's negative helicity, which is spin counterclockwise along the lines of motion. That's not really an antiparticle, but that's that same kind of symmetry between them. What photons don't have is electric charge. So they don't need to have the same kind of antiparticles that electrons have, where you have a positively charged positron as the antiparticle of the negatively charged electron. All of this, the point is, about all of this, is that words like particles and antiparticles are useful for human beings. What matters is what interactions there are between different kinds of fields and their associated particles. So there's an interaction where an electron and a positron annihilate into photons or other particles. So that kind of annihilation process can be thought of as particle, antiparticle destroying each other, right? And the energy going into some other kind of field behavior. For photons, two photons can come in and do the opposite. They can convert into an electron and a positron, okay? So the two photons both disappear. There you go, that in some sense, that's kind of like the photon being its own antiparticle, even though those two photons that came in are the same kind of particle. They're vibrations in the same field, the underlying electromagnetic field. So it's a slightly, it's a case where the language of particles and antiparticles doesn't quite fit comfortably onto the reality of photons, or for that matter, gravitons and other particles like that.
20
2024.0619Genson says, humanity managed multiple times in its history to cause widespread chronic lead poisonings with all their consequences including imbecility and madness. The last time it was due to tetraethyl lead. Many people tried to prevent it this time like Yandle Henderson in 1925. Okay I should have edited this part out sorry. How can we trust our political system or science with anything? Honesty, justice, or fairness? Given that they were unable to handle even such obvious stuff, or was it actually handled well because the poisoning was stopped after all?
Well, I think when you say, how can we trust our political system or science, there's a kind of a question being begged there. Should we trust our political system or science? I don't think that the idea of trusting our political system or trusting science is a good idea. It's more complicated, it's not a bad idea either. It's not that you should distrust our political system or science, it's just that is a far too simplistic way of approaching an attitude towards our political system or science. Our political system, as well as our scientific system, these are vast collections of many people doing all sorts of kinds of things. Systems that will sometimes do very good things, sometimes do very bad things. The idea of simply having an on-off switch for trust. For our political system or for science is I think way too simplistic. We need to think about the situation. We need to understand that there will be conditions, circumstances under which our political systems will systematically fail us. And we need to fight that. We need to be whistleblowers and support other whistleblowers when they exist. We might need to protest. We might need to try to change the system in various ways. Likewise for science. Science is... In some narrow conception, just trying to learn true things about the universe, but of course in a broader conception, it's part of a process of technology and engineering and the real world, capitalism, et cetera, where it can be put to good use or bad use, it can be manipulated in various ways, true scientific findings can be hidden, false ones can be promulgated, etcetera. So you have to be careful about it. You have to be wise to the fact that Experts know more than non-experts do. Experts are not always right. Both of those things are true at the same time. Scientists are generally trying to find the truth. Sometimes they're malicious people who have other agendas. Both things can be true at the same time. You just have to keep your wits about you. That's all I can say.
21
2024.0620Edward Crump says, please give your elevator pitch to better understand emergence.
I honestly don't see what's so hard about understanding emergence. If I have a box of gas with many different atoms in it, I could describe it telling you the state of every single atom. Or it turns out, and this is a highly nontrivial fact, that I could take little regions of space inside the box of gas and consider the average behavior of the atoms. I can throw away an enormous amount of information. I can forget about the specifics. Of the many, many atoms in some little cubic millimeter of space, and instead just tell you the total density, total energy, average momentum, things like that. And it turns out, miracle of miracles, that even though you've thrown away almost all of the information and kept only a tiny little bit of the remnant information, that is still enough to make very, very accurate predictions about the future. That is what Dan Denick called a real pattern lurking in. The coarse-grained dynamics of the system, and that to me is what emergence is all about.
22
2024.0621DMI says, can you explain how the Riesz-Fischer theorem can be true?
So the Riesz-Fischer theorem is this remarkable theorem in quantum field theory, and it's about the vacuum state. It's about empty space. It's one of those things that convinces you that even the vacuum, even empty space in quantum field theory is interesting, even before you start talking about particles and Feynman diagrams and whatever. And roughly the theorem says that if I have some region of space in the vacuum of quantum field theory, there are things I can do to it, essentially observations I can make, such that after the observation is made, depending on the observation outcome, the rest of the state, things many miles or light years away can be literally anything, or arbitrarily close to literally anything. This is sometimes called the Taj Mahal theorem because one way of making it vivid is to say that there's a measurement I can do here in a laboratory here on my basement that after doing it and getting a certain measurement outcome there's now a Taj Mahal, a copy of the Taj Mahal on the moon or on the moon in a different galaxy for that matter. This seems very strange to people because it's the vacuum state, it's empty space. It's the emptiest that space could be but that's all because of quantum mechanics and quantum field theory being subtle in various ways. So to understand where the theorem comes from, at least at the hand wavy level, is there's sort of two steps. One is to think about entanglement, okay? So if you've heard the conventional story of EPR or Bell or whatever, where you have two particles with spins, spin up and spin down, and you have Alice and Bob, and Alice has her spin, and it could be in a superposition of spin up and spin down. That means that she doesn't know what she's gonna get when she measures spin up and spin down. She's gonna measure it along some particular axis, right? She's gonna set up a magnetic field in the z direction and then measure the spin along that axis and the particles in a state where it's a 50-50 chance of getting either one. But it can be the case that Alice's particle is entangled with Bob's particle and the entangled state has the property that Bob, who could be a million miles away or in Alpha Centauri. Also doesn't know what answer he would get if he measures his spin in the z-axis also, but they're entangled in such a way that we know the spins are opposite. So whatever Alice gets, we know that Bob's gonna get the opposite one. That's just the usual EPR story. Now there is another layer there, still talking about the first part of the Riesz-Fischer theorem. There's another implication of this, which is Alice didn't need to measure her particle along the z-axis. Axis, she could have measured it along the x-axis or the y-axis or any combination, any diagonal axis, okay? She will always get, by the rules of quantum mechanics, one of two possible outcomes, either spin up or spin down with respect to the axis along which she's measuring it, okay? And it is something you can show using the mathematics of quantum mechanics that when she gets a measurement like spin up along the x-axis, maybe that's not a good one to pick. When she gets a measurement of spin up along some axis you can always, with a set of measures zero, you can make a measurement so you get 100% chance of getting a certain outcome and 0% chance of getting the other one. So forget about that very, very unlikely possibility. For a generic choice of spin axis, whatever answer she gets, we know that Bob has now a spin which is aligned exactly the opposite, okay? So it's not. Just that Alice can measure spin in the z direction, and when she gets her measurement outcome, now we know Bob has the opposite spin. It's even when Alice measures along some other axis, it's still true that Bob's spin is now pointed in the opposite direction along that same kind of axis. So in a very real sense, there is a possible measurement outcome Alice could get by doing all sorts of different measurements. She only gets to do one, but it's possible she would get an outcome. Such with the implication that Bob's spin is now in almost any situation we could want, almost any state, depending on Alice's measurement outcome. Crucially, Alice can't choose what measurement outcome she gets. So the theorem is not Alice can force Bob's spin to be in any state, it's that there exist measurement outcomes that Alice could see that would imply that Bob's spin is in any state now, okay? So that's one feature of entanglement. The other aspect of the Riesz-Fischer theorem is thinking about quantum fields. And when we think about the vacuum state of quantum fields, we often think about a wave stretching through all of space or many, many waves with different wavelengths stretching through all of space. And we think of we build up the space of all possible quantum states as superpositions of all those things. And we ask, what is the lowest energy state? And so there's an answer. There's a unique answer. What is lowest energy state globally to the quantum field theory. You can't see, because this is audio only, but I'm waving my hands to indicate that we are stretching throughout all of space. Alternatively, we could look at one region of space and another region of space separately. And that's just another way, it's basically another choice of coordinates in the space of fields, okay? So we can talk about what the... Modes of the field are doing in region A for Alice and separately what they're doing in region B for Bob. And that's not all of space. There's all sorts of other regions of space where neither Alice or Bob live, but let's just look at those two regions. In each region, given that overall we are in the vacuum state of the quantum field theory, there is a superposition of many different states of different energies in each region because it is, because of entanglement. And not just entanglement, but because of interactions between the quantum fields. Like, let's think of it this way. I know there's an overly long explanation, sorry about that. But think about a situation where I once again have two spins, like Alice and Bob did. So I'm not thinking about quantum field theory. I think of spins again. But now there's a magnetic field. So the magnetic field's in the z direction. So it's in the sense that the energy of the spin is a little bit less when the spin is pointing in the z direction than opposite to the z direction. Okay, so there's two different energies for spin up and spin down. So left to themselves, both spins would like to be in the lowest energy configuration, spin up. But imagine that I bring the spins together and there's an interaction between them, which says that it's more energy when the spins are the same, and less energy when the spins are opposite, right? Now, there's two forces pushing them in opposite directions. One force says, by themselves, they wanna be spin up. Along the magnetic field, but because of the interaction between them, they want to be opposite, they can't both be spin up and opposite. So the actual ground state of that system will be a superposition of spin up and spin down for each of them, and also spin up and spin up and spin down, like all of those possibilities will be superimposed in exactly the right way to make the ground state of the system. Okay, the same thing is true in quantum field theory. If I divide space into different regions, I can't just separately put each region in its lowest energy state and think that that's gonna be the lowest energy state overall. There are interactions between the fields next to each other that will relate what they're doing. And so if I know that overall, the quantum field theory is in its lowest energy state, that is a superposition of local things in each region, which are not in their individual. Lowest energy states, okay? So all of the things that the field can do in any region actually contribute to the overall vacuum state. And, I know it's a long answer, sorry about that, and all of the regions are entangled. What the fields are doing in every region of space is entangled because of features of quantum field theory. That is what, just like with the spins, that is going to turn out, due to the dynamics of quantum field theory, to be the lowest energy state. So basically, the little fields that you're thinking about as localized to the region in front of Alice have a state that is entangled in all of their different possibilities. Like everything the fields could be doing in front of Alice are contributing to this entangled state with everything the fields that could be doing in front of Bob. And therefore, when Alice makes a measurement in her region. There is a chance, an arbitrarily small chance, generally very, very small chance, but still a chance that her measurement outcome will imply that Bob's state is literally anything, almost anything, there's different little mathematical footnotes there, but essentially anything, including a picture of the Taj Mahal. Again, she can't force it to happen, but there is so much entanglement in the vacuum state of quantum field theory that those things are always going to be possible even if they are very, very unlikely.
23
2024.0622Whoo, Jim Murphy says, as I've been listening to and participating in AMA episodes over the years, the number of questions has increased dramatically. Obviously this is good for you, but I can't help but feel a little bit selflessly frustrated by the sheer number of questions that have been answered many times over. I would gladly pay more money for content discussing deeper technical points. Would you ever consider something like this?
I'm certainly open for Patreons to ask for or suggest ways to... Organize the AMAs differently. I think I don't want to do, I think there's a consensus that having one AMA per month and the other episodes per month be conversations with different smart people is a good balance overall. But right now, the way that the AMAs work is, I just try to pick the questions I can give the most interesting answers to in my own personal opinions. If there's some other way that there's a consensus that AMAs could work, then I'm happy to do that. But I would. I would guess not. I think that I know that different people who are very kindly supporting the podcast have different sets of questions and different levels of questions and different goals in asking questions. Thinking of the AMAs or the Mindscape podcast in general as a physics tutoring system is probably just going to lead to frustrations. That's not what we're here to do. Off the cuff answers from me to various kinds of questions. They're not systematic, they're not overly pedagogical, they're not very deep, and I suspect that's not what they're going to be. If there's some topic that everyone on Patreon is really, really interested in hearing, then maybe just suggest a solo episode, and that is something I would absolutely think about doing. Also, I should mention that... Yeah, this month in particular, we had a lot of questions asked in AMAs where I have answered them before. And so I'm trying not to answer them here. And I feel bad because they're perfectly good questions, but I've answered them before. So I'll remind people, we have a deep archive and we is completely searchable on, certainly on prep and also on Patreon. You can look for the possibility that you have a question that's already been answered. And even if not. You can listen to AMAs from months past, maybe that would be fun to do.
24
2024.0623Chris Murray says, in quanta and fields, each plane wave mode of a field configuration is described by a height variable and a wave vector, but phase is not mentioned. Does the phase somehow not matter, or is it contained in the height variables, a complex value, or what's going on?
Very briefly, yeah, in the book, I frequently glossed over complex numbers. In particular, the way that Fourier transforms work, that is to say, the way that you switch from discussing a function by giving its value at every point to giving the different contribution of waves of every wavelength, inevitably involves complex numbers. Fourier transforms are generally complex valued, even if you start with a field that is completely real. So that happens all the time in quantum field theory, there really are complex numbers there. We just didn't talk about it in the book because it was sort of not centrally important for the physics that is what we cared about at the end of the day.
25
2024.0624Peter Becker says, I've been wondering whether there is some good philosophical reason why the smallest things in the universe, say extremely high energy photons, could have as much energy as the largest things in the universe like stars, black holes, et cetera. Is there any good reason why the shorter the wavelength of the photon gets, the more energy it has?
Well, there is a good reason for that. I'm trying to figure out a way to say it that is not just repeating it. In wave mechanics generally, okay? So forget about quantum mechanics specifically, think about electromagnetism, or for that matter, waves on the ocean. If you have waves of a fixed amplitude, so the fixed height of the wave going up and down, as the wave length gets smaller, the wave has to change more rapidly, right, to go up and down in a shorter distance. That takes more energy, that is a higher energy configuration and that's just as true in quantum field theory as it is everywhere else. So the general rule is that higher energy things have shorter wavelengths, that's just always true. The reason why it seems counterintuitive is because in the classical world we don't care about the wavelengths of things. When I think about a dumbbell or the earth and I think about how much mass they have. Those systems have a center of mass coordinate that has a wavelength in quantum mechanics, but that wavelength is absurdly tiny because the systems are very massive. As we talked about before, when you have very, very massive systems, both position and momentum, sorry, both position and velocity can be very highly localized because mass is very large. So what we think of as the size of a dumbbell or the earth or whatever, is its physical extent in space classically, not its quantum mechanical wave function. So for classical things, heavier generally corresponds to bigger because there's more mass there, there's more stuff that can contribute. It's only when you get to sufficiently small things that you have only one particle that there is a different kind of effect that kicks in and you can't make things smaller than their Compton wavelength, otherwise you start making more particles. So it's just there are different reasons why the energy is going up in the classical regime and the one particle quantum regime.
26
2024.0625Marcin Chady says, as a physicist and wine aficionado, do you consider it a sacrilege to bring red wine to room temperature when a previously open bottle has been stored in the fridge overnight, say, by heating it up in a microwave?
I am not religious, so it's not sacrilege, but I certainly would never do it. You run a danger if you microwave wine of changing its delicate composition and chemistry Even though you're trying not to I don't actually know maybe it's perfectly okay Maybe if you do it just a little tiny bit, it doesn't matter that much but I will mention two things number one I'm completely happy with Chilled red wine. That's fine. In fact, I think that it's a little bit of a myth to think that you need to serve red wine at room temperature. It depends on the wine. Different red wines respond differently to different temperatures, but a little bit lower than room temperature is entirely appropriate. And the wine sort of behaves better in storage if it's a little colder. Yes, it is true that in the ancient days, they would not have refrigerators when they stored their red wine, but they would have dank. Basements in the chateau which got pretty chilly and that's what the temperature which you would serve your wine So serving wine that 60 degrees or 65 degrees Fahrenheit is actually good and we keep our red wine in a Special wine fridge. We have a little tiny wine fridge in the kitchen and We keep the wine at 55 or 60 degrees so that when you bring it out and let it sit there for a little while over the course of opening it up and drinking it goes from 60 degrees to 70 degrees or whatever, and that goes through the range of the best possible temperatures. So that's what I would advocate doing. Oh, and the other thing is, I don't think you have to store the wine in the refrigerator. It's much better, again, I think this is true, I'm not a super expert, but much more important to remove the oxygen from the bottle. If you have a half, if you have a half full wine bottle, so you open the wine bottle, you... Do not and you should not feel obligated to finish all of the wine. You drink as much as you want, but then it will go bad if it's exposed to air for a very long time. So you can buy a very cheap device called a vacuum bin, which will basically vacuum pump the air out of the wine bottle and then put a little rubber stopper in it, okay? That actually works, at least for a day or two of keeping the wine. It wouldn't work for weeks, et cetera, but if you do that, you don't have to refrigerate the wine.
27
2024.0626Alan Lubell says, I listened again to your Max Tegmark interview and really enjoyed it. I'd like to ask when Max says, if I make a measurement of a particle that is in two places at once, and if in advance I've decided that if it's here I'm going to go for a drink and if it's in the other place I'm gonna watch Netflix, is he saying that in order for branching to occur you have to be making a decision or choosing a certain action and not just sitting around doing nothing?
No, he is certainly not saying that. And notice, he did not say that. He said, if I made a measurement And in advance I decided to react in certain ways when that measurement is made. What he's doing is not branching the wave function of the universe by his decision, he's branching it by making a measurement of a particle. That's when the branching happens. What he's doing is making the two branches macroscopically different by deciding ahead of time to behave in two different ways depending on the measurement outcome. So very often if you have a single radioactive particle that decays or a single spin that is measured or whatever, in principle, you branch the wave function of the universe into two universes, but who cares? They're indistinguishable for all macroscopic purposes. You don't notice most of these branching events. So what Max is just doing is a Schrodinger's cat-like thought experiment where he's saying, I'm going to figure it out. I'm going to set up a system where the two branches truly are different macroscopically.
28
2024.0627Griffin Kistler says, with your accomplishments and position, do you encounter imposter syndrome? If so, do you have any strategies you could share that would be helpful in mitigating it or navigating it with the assumption that your advice is directed at a Sean Carroll of another discipline?
I think we all have our psychologies with positive aspects and negative aspects. Imposter syndrome has generally not been mine. I've certainly been intimidated by talking to people who I... In other words, I think I've had imposter syndrome when I deserved to have it, when I really was an imposter, let's put it that way, but less, not so much when I didn't deserve to have it. I think that's true, roughly speaking. Again, one always makes mistakes. Sometimes one errs on the side of being too arrogant, sometimes of being not arrogant enough. I just think you have to recognize the difference between your inner psychological state and the objective external reality. It's very, very possible to be able to say, I objectively know that this is not true and nevertheless I feel like it is, right? I act psychologically like it is. I think that's what you have to work to overcome, to both to objectively understand where, people can be very annoying when they act like experts but aren't. That's a known annoying thing. It's not quite as annoying but also a failure mode. To not recognize when you are an expert or accomplished even though you are. It's hard to correctly calibrate that, but try to do it, try your best to do it. That's all I can say. I don't think that's very helpful advice, but I think ultimately that's what we gotta try to do.
29
2024.0628Casey Mahone says, "'Why do people say that we can never reach absolute zero? Wouldn't that just mean that a molecule is sitting completely still? Of course the wave function is still evolving, but I would have thought the temperature referred to the motion of the molecule through space.
Well two things. One is, I think when people say we can never reach absolute zero, it's arguably a bit of an exaggeration. I think that as a practical issue, that is certainly true. I can imagine a single molecule or a single particle that we put into its ground state. To a very high confidence anyway. Maybe not to perfect confidence, maybe that's what people mean by not reaching absolute zero, but we can get arbitrarily close. It's very, very difficult to do that for a whole bunch of reasons. We don't have control over the way in which systems cool down. Like if systems are cooling down by emitting photons, you could in principle. Emit them, if there's a smooth continuum of states as you approach zero energy, then you can get there in a series of steps that would take you infinitely long to get to absolute zero. And moreover, the world is a noisy place. You and I are emitting a whole bunch of radiation in infrared wavelengths. So if you just yourself in front of you in your eyeballs have a system you're trying to get to absolute zero. The fact that you yourself are heating it up becomes a real problem. So I think there's all sorts of practical issues that are more relevant than the theoretical impossibility that is sometimes implied. But the other one that is very important is that you shouldn't talk about absolute zero for a single molecule. You shouldn't talk about temperature for a single molecule. Temperature really only begins to make sense when you have many, many molecules and you reach some kind of thermodynamic limit. And in that, case, it becomes all those issues we discussed as making it hard to make a single molecule in its ground state become even more difficult because you have many molecules with relative random motions and it becomes all that harder to get rid of all the energy in that system. So I think that we can contemplate systems in absolute zero temperature, but we're never going to actually make them.
30
2024.0629Zach McKinney says, in your recent solo episode on the coming transition in how humanity lives, you spoke about the possibility of a coming phase transition in human society with reference to Jeffrey West's recent lecture showing evidence of an approaching technological singularity. What physically observable quantities do you believe will be the most fundamental to recognizing or characterizing such a phase transition? In particular, are you thinking of this phase transition primarily in terms of economic metrics, such as rates of technological innovation adoption? Or do you think there are also important social metrics that may be used for identifying characterized radical changes in the dynamics about humans behave?
Both. I think that one, and it could be wrong, of course, but under the analogy with physical systems, you generally have multiple measurable quantities that undergo phase transitions at the same time. Really, a phase transition is sort of a... Change of the collective behavior that you get from all these individual constituents interacting with each other. The difference between solid and liquid water, okay? There's a lot of differences. One, of course, is the viscosity. I guess you wanna put it that, the solidity of it. But also things like the equation of state, the speed of sound, all these things, the density, they all change at the same exact point. And I suspect that under, for good reason. I suspect that the analogy works for human societies as well, and if there is going to be some phase transition, it'll probably be at more or less the same moment for more or less different kinds of metrics, both economic and social.
31
2024.0630Jeffrey Clark says, it seems the very foundation of modern cosmology is the Hubble law of expansion. With a good telescope, we find galaxies containing heavier elements, several generations of stars, with redshifts which place them observed less than 300 million years after the Big Bang. If we pretend we are ignorant of the expansion caused by the redshift, might we solve some of these quandaries, I.e. The sage, dispersive medium, etcetera. Did Hubble send us down a rabbit hole we are afraid to crawl out of?
No, Hubble did not send us down a rabbit hole we are afraid to crawl out of. What is important to understand here is that Hubble discovers the relationship between apparent velocity and distance of galaxies back in the 1920s. And then you say, okay, the universe is expanding. But since then, in the almost 100 years since then, there's been enormous and completely independent lines of evidence that support the same idea. Not to mention the fact that general relativity predicts the same idea. But we have nucleosynthesis, we have large-scale structure formation, we have the microwave background, we have various... Concordances between ages of stars and galaxies and the universe and the whole bit, there's no way of getting out of the overall Big Bang model. So that is absolutely solid. I don't think that there's evidence that there's been several generations of stars in very early galaxies, but it's true that we have better telescopes and we discovered galaxies that are very early in the history of the universe. That's a challenge for galaxy formation. It is possibly a challenge for star formation. Two things we don't know a lot about. It is absolutely not in any way a challenge for the overall Big Bang model or the Hubble expansion.
32
2024.0631Kyle Cabezares says you mentioned you were open to giving dating/relationship advice in a previous episode so here's a softball. In your opinion what is the biggest red flag and biggest green flag when choosing a romantic partner?
I think that these kinds of questions are just too vague. I think that there's no useful answer to a question like that because people are different. I know I end up saying this all the time but people are different. Different kinds of relationships will work. Different kinds of romantic partners will work depending on your particular needs or incompatibilities. So I think it's a mistake to think that there is some list of red flags and green flags that is universally applicable to all sorts of different kinds of people. In fact the real difficulty in dating sites and things like that which in general I'm in favor of I think there's lots of different ways to meet people. No reason why not to use technology to do it. But the idea that you have some kind of checklist that here is what you want in a partner and here's what you don't want in a partner and you're gonna find partners by matching and comparing checklists is just completely contrary to all evidence. As well as if you think about the problem there's no reason to think that should work. You have to talk to people you have to meet them you have to interact with them and you have to get a feeling for how you respond to them before knowing whether it works. So rather than thinking in terms of red flags and green flags I do think that you can trick yourself you can fool yourself. I mean I know I have I'm sure many other people have I'm sure that there are things that other people do that your potential romantic partners do that should lead you to realize that they are not good romantic partners for you that you convince yourself aren't actually that bad. So I think you should look for the red flags in yourself but not really red flags so much as blind spots. It's an interesting question because of course there are things about people that might get in the way of a successful relationship but are changeable are fixable and there are other things that are just intrinsic to who they are and who they're not going what they're not going to fix. So and that's a difficult thing to decide about. I think that the maybe again in the along the lines of think about yourself more than fixing or finding flaw in the other person if you find that you are embarrassed about certain things that another person does or you're making excuses or when you tell your friends about your partner and you find that mostly you're complaining then that kind of thing is a red flag for the relationship. You should mostly be happy in a successful relationship. If you find that you're constantly making excuses if you find that they're not trying to make you happy if you're trying if you are in love with the idea of who they might be if they were a little bit different than who they actually are those are all relationship flags that you should pay attention to not so much flags about that individual person.
33
2024.0632All right I'm going to group two questions together. One from Joseph Dundee regarding your recent episode Excellent Podcast with Cumrun Vafa what do you think is the likelihood of finding a deviation in Newton's inverse square law at the micron range? I read that such a result either confirming or disproving a deviation should be obtainable experimentally within the next three years or so. And Michael Wall says regarding your discussion with Cumrun Vafa are you able to comment on the idea of a dark dimension with length of order 10 microns? From what I gather only the modes of the graviton live in this dimension but where does that leave standard model particles? How is it possible to do any experiments if standard model particles don't move in that dimension?
Yeah the whole idea which started back in the late 90s from a bunch of different people talking about it I think that who were the authors on the paper? Dimopoulos, Dvali, Arkani Hamed. I'm going to leave people out. But the idea came from the realization that in string theory there are higher dimensional branes B-R-A-N-E-S as well as one-dimensional strings one-spatial dimensional strings. So imagine that all of space was not compactified right? Imagine that there were nine dimensions of space that were just infinitely big but embedded in that space there was a three-dimensional brane okay? A three-brane. So it turns out that various fields can be confined to only propagate on the brane. So if you have fields like quarks and electromagnetism and other forces that we know and love from the standard model but that are stuck on a brane that is three-dimensional then the world would appear effectively three-dimensional to you even if it were embedded in higher dimensions. The one exception to that is gravity. Gravity is a feature of the curvature of space-time and therefore you cannot confine it to a brane. There are slight exceptions to that. Randall-Sundrum warped extra-dimension models are slightly different than that. But basically the idea is that you could have extra dimensions where only gravity propagates. So as long as you're not asking questions about gravity everything looks like it's three-dimensional. Now that's interesting because gravity is harder to experimentally probe than other forces because it's so weak. We say we're probing gravity at scales of millimeters or microns or whatever that sounds pretty good but we're probing electromagnetism and the strong and weak nuclear forces at much much much smaller length scales than that precisely because they're stronger and easier to probe down there. So it is plausible that one or more extra dimensions are bigger than a micron or well bigger than particle physics distances okay? Certainly much bigger than the Planck scale. How plausible is it? What is the likelihood? I don't know. I honestly don't know. There is the original motivation back in the late 90s was that if you had two extra dimensions then you could understand why gravity seems so weak in our macroscopic world because it sort of is diluted by the large extra dimensions. They would be you could actually have the Planck scale the real Planck scale which we normally think of as much much higher than particle physics energies you could actually have it quite close to the electroweak scale that was being probed by the Large Hadron Collider if you have specifically two extra dimensions. Why two? Well because there's a relationship between how big the extra dimensions are and how weak gravity gets. And there was an interesting coincidence that two dimensions could affect the force of gravity so that the Planck scale is at the electroweak scale and the two extra dimensions are large enough to be experimentally testable. But they did the experimental tests and they didn't find any evidence of two large extra dimensions. So if you have one large extra dimensions like Vafa was talking about that would not have this unification as far as I understand it between the Planck scale and the electroweak scale. It would be a different kind of thing which is necessary because we turned on the LHC and we didn't find gravitons or anything like that. But in the 1990s that was a thing to think about finding gravitons at the Large Hadron Collider. It didn't work it didn't happen not yet anyway but that was something to think about. So I think it's something to think about. I wouldn't put it individually plausible. I think there's lots of other possibilities out there. I shouldn't say plausible. Likely I don't think it's individually likely. It's completely plausible okay? So I think we got to be open-minded because we're a little stuck right now about where to go beyond the electroweak scale. So this idea seems as good as any to me.
34
2024.0633Voltaire O says if you were to have a tattoo what would it be? Assuming you don't already have one.
I don't already have one. I don't have any plans to have one. Getting a tattoo seems to be like a bargain you make with your future self that you'll still find it amusing to have one when you're 20 years older than you are now. I'm not quite sure I have the confidence to do that but also I don't have any special desires to get a tattoo. I've joked that for people of my generation who played Dungeons and Dragons when it first became popular circa the late '70s early '80s I do have a sentimental fondness for those old illustrations from the Dungeons and Dragons books. There's all these crazy pictures of monsters and things like that slightly amateurishly done but often with something of a sense of humor. So there's this wonderful drawing of a lurker above that was one of the monsters in the Monster Manual. It would disguise itself as a ceiling. It was kind of a flat pancake-y thing the lurker above. And it would disguise itself as the ceiling of the room you're in and then it would fall on you. And there's this great drawing in one of the books of lurker above falling on a poor warrior with their look of surprise and their sword drawn. That's a tattoo I could imagine myself getting. It's not going to happen don't worry. But that was the joke that I was happy to go along with.
35
2024.0634Samuel Benjamin says I've been a longtime listener to and patron of Mindscape and love the AMAs. The podcast you did with Joe Rogan however still stuck in my mind as some of the most effective communication on quantum mechanics many worlds and other typical Mindscape topics by you or anyone else. I think it was being asked questions by a layperson that made it so clear and engaging. Plus Joe for all his myriad faults is not afraid of asking stupid questions. My question is have you ever considered doing something like this set up on Mindscape or on someone else's podcast where you answer a layperson's question on a given topic?
Well I did something like that when Something Deeply Hidden came out. Rather than doing a solo podcast where I talk about many worlds I did an inverse podcast with Robert Reed where he asked me questions. He was a little bit obviously more expert than Joe Rogan was. I don't know what the answer is to the specific question but let me mention a certain reality of the situation. The audience for Mindscape is going to be full of people with different levels of knowledge right? Some are going to have heard a lot about quantum mechanics already. They don't want to hear the same thing over and over again. But many will be absolutely new to it and they will not have heard anything at all. So it is difficult to keep everyone happy at once. And I try in the AMAs for example to mix things up. So some people get frustrated because there are somewhat technical questions like about the Riesz-Fischer theorem. Other people get frustrated because there are basic questions like how many times per second does the world branch or something like that. And I have to try to guess from the question what level of answer would be appropriate. Sometimes I guess right sometimes I guess wrong. There's no perfect solution to this right? Someone else asked just earlier in the AMA could you have more technical deeper things going on in the solo episodes or the AMAs? There's no one right way to do it. So I try to do different things at different levels for different audiences in different circumstances. And I do a lot of things but I can't do them all. So let me know through various methods if you think that some way I could be doing it better. I'm always happy to take suggestions.
36
2024.0635Now I'm going to group two questions. Actually the last question these next two are kind of grouped together but they're separate. So I'm grouping just these two having to stand to the last one. Shambles says I recently listened to a debate between the archaeologist Flint Dibble and journalist/pseudo-archaeologist Graham Hancock on a popular podcast. It's Joe Rogan's podcast. You don't have to hide that. We're allowed to say it out loud here. Unsurprisingly the Hancock fans hailed this as a great victory for him and his wacky ideas despite the total lack of evidence to support them. Be it this or other debates between experts and conspiracy theorists or science deniers do you think these public exchanges are useful or do they just give oxygen and legitimacy to nonsense? And then Cole Justo says recently a clip of Terence Howard on Joe Rogan went viral on Twitter. In the clip he claims that the periodic table is wrong and that a better version was revealed to him in a dream when in his mother's womb. This is obviously crazy but many people online seem to want to hear him out and defend him against legitimate scientists who made their best efforts to debunk it. In general the anti-establishment sentiment that scientists are hiding some deep truth about the universe and are untrustworthy just bummed me out. Do you have any ideas for ways to rebuild the trust between our institutions of knowledge and the common citizen?
So both questions have to do with how one deals with bad science in public fora such as Joe Rogan's podcast or anywhere else. And I think that once again not to be wishy-washy about it but it's going to depend on the listener what kind of strategy works the best. You have to ignore the fact that some people are just going to believe nonsense no matter what you do to them right? You can't be frustrated by that. It might be frustrating might be annoying but that doesn't mean there aren't other people who you could reach by some effective methods. You have to think about and strategize focused on the people who might be persuadable not the people who are already lost to reason and common sense okay? So I'm not going to try to convince Terrence Howard. I might try to convince someone who is mildly curious about Terrence Howard and likewise Graham Hancock and his fans. Once you're a deep fan of Graham Hancock then I'm not going to try to convince you I've moved on. But if you've never heard about it before and maybe you think that it's potentially interesting then I think it's very very useful that someone like Flint Dibble will take the time to say no here is why real serious archaeologists don't talk this way okay? But you can also overdo it. If you spend all of your time debunking nonsense then you don't have any time to get out the good stuff. So as part of thinking that there are different strategies and different ways to communicate good science and build trust and things like that my own personal preference is to talk about the good stuff to lead by example if you want to put it that way. I don't want to spend my time debunking nonsense. I've done it in the past. I'm happy to do it when the occasion calls for it. But I don't want to spend most of my time doing it. I think that in our urge to debunk nonsense we very often don't have time to talk about all the cool stuff that's going on. So Mindscape in particular is devoted to talking about the cool stuff that is going on. And I know that Mindscape is never going to be the biggest podcast out there it has a very good sized audience by my expectations when I started it but it's never going to be number one on the charts or top 10 on the charts or anything like that. It's for the audience that wants nothing but the good stuff that is not attracted to the conspiracy theories the wild alternative pseudoscientific nonsense things like that. And those people deserve to hear even more good stuff. That's what I'm all about and that's what I'm going to spend most of my time doing.
37
2024.0636James Allen says when reading your books on Kindle where can we see we can see where other people have made highlights. As the author do you ever go through and see what other people are highlighting to see what parts are resonating particularly with readers?
Not so much to be honest. I have seen it because I do download Kindle versions of my own books just so I can have them on my iPad which is where I read Kindle and it's very good as a resource sometimes. And then you notice when there are underlinings and things like that. But what I've noticed is it's not there's no relationship between the things that I think are most interesting and insightful and the things that get underlined by readers. Because if I'm writing a book on something I've thought about it a lot and read about it a lot and am more knowledgeable about what has been discussed over the years. And what people underline is often like the most obvious stuff or the things that are has been established for the longest and so less interesting to me. So you got to say it you got to say the things no one is born being an expert in anything. So it's the first time that you learn any fact that's crucially important. I try to do a good job at explaining the well-known facts to people but it's sort of not the part that is most interesting to me of course writing the book. So I'm happy when people underline things but it doesn't affect my writing or reading very strongly.
38
2024.0637Matthew Wright says what are some of the cutest things that Ariel and Caliban have done recently?
Ariel and Caliban do a lot of cute things all the time. We were just on a trip to France for 10 days. So we had a cat sitter come in every day and visit Ariel and Caliban. And they're both especially Caliban well they have different personalities. Caliban is very social. He wants to be with people. When people come over he visits them and says hi. When we're here he wants to be close to us at all times. So he is just a love monger. He wants attention. And when we're gone for a long time he gets annoyed by the fact that we're gone. He doesn't like it. Ariel is much more of a delicate flower and she doesn't if there's people over she's hiding. People don't see Ariel. Our cat sitter is now a regular cat sitter. So she's made friends with the cat sitter. So she'll come out for her. But most other people she's not gonna try to socialize with. But she's also needy. She wants she sleeps in the bed with us at night. She needs that time. She has special places around the house where she will go to to get petted and things like that. And after you come back from a long trip she wants extra pets and extra reassurance that we're here we love her we're not going to abandon her. So it's not specific cute things that they've done recently so much as just they've been wanting reassurance that we are our absence for 10 days is not the sign of anything to come. And so we've been giving them that.
39
2024.0638Jonathan Good says does eternal inflation have an anthropic preference for young universes since they are most recently undergoing inflation? If so could it be used as an argument against Boltzmann brains?
Well this is gesturing toward an important unclear problem in eternal inflation which is called the cosmological measure problem. If you have in so not to spend too much time explaining eternal inflation but basically in many models of inflationary cosmology which makes the universe expand at a super fast rate at early times in some region of space inflation ends and the universe gets hot and it looks afterward like the Big Bang looks like the kind of universe we live in. But in other parts of the universe inflation keeps going and it keeps going forever. So in an infinite amount of time you make an infinite amount of universe and an infinite number of different things go on. And now of course you want to make predictions in that kind of universe. And it's hard to do because an infinite number of things go on. And if you want to say well what is the ratio of A happening to B happening A and B are both going to happen infinitely many times. So you're dividing infinity by infinity and that's hard to do. That's the cosmic measure problem. I don't think it's been solved. I don't think we have any decent way of answering these questions. And I think that people kind of under-emphasize this problem. I mean there are people who are over-emphasizing it also. People like Paul Steinhardt who is a very thoughtful guy one of the pioneers of inflationary cosmology. I think he's gone too far to the other side. He will say now that because of these infinities in eternal inflation inflation makes no predictions whatsoever and is completely non-scientific. I think that's going too far. I think we have to think carefully about how to make predictions under these circumstances. And guess what? I think it's the perfect kind of thing for philosophers and physicists to get together and work to try to solve. They haven't done that so I don't know what the correct predictions are. There are plenty of attempts that make silly predictions and some of them do have what is called the youngness paradox. But I do not think that that actually has any relationship to reality. Neither way no approach that I know about helps solve the argument with Boltzmann brains because as soon as inflation does end in any one part of the universe you don't know when you are in the universe much less where you are in the universe. If you're just asking where could I possibly find myself any part of the universe if there is a possibility of Boltzmann brains existing eventually the Boltzmann brains dominate. There's going to be much more Boltzmann brains than not. It's very hard to wiggle out of that by making predictions carefully in some inflationary scenario.
40
2024.0639Matthew Cushman says listening to your recent solo episode on AI and your interview with Gavin Schmidt it seems to me that you would be concerned with both the environmental and animal welfare impacts of food. What are your thoughts on ethical eating e.g. Veganism vegetarianism pescatarianism meatless Mondays or whatever?
I think that Well number one the environmental and animal welfare impacts are both perfectly legitimate things to talk about but two very very separate issues. I wouldn't want to get them mixed up with each other. But more importantly in other contexts I've said this and in this context I think I said this too I think that it's a mistake to address these issues as concerns of personal virtue that you are making the world a better place by not eating the hamburgers or whatever. I mean maybe it's true but it's such a tiny tiny impact on the world that I want to roll my eyes and say you're not really helping the world that much. The way to help the world is to change the system okay? If you think that and it's true as we talked with Hannah Ritchie for example meat beef especially exerts a disproportionately bad impact on the environment and therefore it would be better if we ate less beef. Even if you have forgetting about the ethical questions even if you thought that it was fine to eat beef but the environmental impacts are bad then eating less would be a good idea. I personally don't eat that much but I eat some a little bit. And but to make the world a better place what you should do is make beef more expensive make it less attractive to eat beef systematically for everyone in the world. That's how to make the world a better place. For example you could make alternatives. I'm a huge fan of meat alternatives in various ways. I think that's the right way to make to actually have a better impact on the world.
41
2024.0640David Wright says your guest Cumrun Vafa surprised me by dismissing supersymmetry as no longer an acceptable research area. Can you identify the research that has led to his conclusion? I'm aware the LHC has not found the LSP yet but has the entire theory been ruled out.
So I'm not sure exactly what you're referring to but certainly Cumrun Vafa of all people does not dismiss supersymmetry as an acceptable research area. String theory relies on supersymmetry. You need supersymmetry. But what he was referring to I think again I'm not exactly sure what quote you have in mind here but what he's referring to is the fact that at low energies very low energies well below the Large Hadron Collider the energies of the ordinary world where we see electrons and protons and things like that there is no supersymmetry. There's no supersymmetry manifest. There is not a particle with equal mass and charge to the electron but a different spin. There would have to be such a particle if there was unbroken supersymmetry at low energies. I think that's all that he was saying. We know that supersymmetry is not manifest in the low energy world. It could be broken and then it would only be manifest at higher energies. We were very very hopeful that that would be the Large Hadron Collider. We would see supersymmetry by now but that turned out not to be the case at least not yet. But string theorists definitely need to be optimistic that it's broken at some energy scale that it exists at very high energies. So and Vafa in particular has done an enormous amount of work on supersymmetry so he does not dismiss it as an acceptable research area.
42
2024.0641Soz S-O-Z asks a priority question. I'm wondering how one can argue for the passage of time. I can see how it is possible to argue for an asymmetry of time. Assuming this asymmetry we can arrive at a B-series convention of time to use McTaggart's terminology. So event A before B but no spatially extended now. Yet what alludes me is how one then ascribes a dynamic notion of passing or flowing to this order of events. You alluded to the possible misconception of thinking of time as flowing in Chapter one of From Eternity to Here. However could we think of passage of time in a sort of B-series way?
I do not like the A-series B-series distinction. I think that that is sort of not a useful way of thinking about time. But I am an eternalist a block universe kind of guy. I think that the best way of thinking about reality is as or at least classical universe reality is a four-dimensional spacetime where all moments of time are just as real as each other just as we ordinarily think of different points of space as just as real as each other even if we're not there. And if in that case there is no physical process called the passage of time all elements of time exist. So the question you want to ask really is why do human beings find it useful to speak a language of time passing? And I think that eventually comes down to entropy increasing and the arrow of time and in particular how that increase of entropy affects our human scale perception of the world evolving through time. Roughly speaking what's going on is that human beings in their minds have images of what they are doing right now of what they were doing a moment in the past and what they will be doing a moment in the future. And they are constantly updating because the moments are different right? Each moment you have a different idea of what you're doing now what you're doing in the future. And there's an imbalance between how you think about that past and how you think about that future. And that constant updating is what gives rise to a feeling that time is passing. It's not a feeling that bears closer examination right? Because time is passing with respect to what? With respect to time that's not kind of a meaningful thing to do. But I don't think the passage of time is a fundamental thing. I think it's a way that we talk and it's perfectly respectable to ask why we find it useful to talk that way.
43
2024.0642Josh Charles says I'm wondering if I misunderstood something about the Boltzmann brain hypothesis. If a Boltzmann brain did emerge from randomness wouldn't there be a preference to sticking around rather than randomly fluctuating back out of existence because of the interactions that happen once the configuration is in place? It feels like there's a preference for higher order complexity because once it appears it's by definition more resilient against random fluctuations.
Well no no that's not true. There is no preference for higher order complexity in thermal equilibrium. All of our intuitions about how the world works are trained on a world where there's a very strong arrow of time because there's a very low entropy pass from which we are still evolving. So our guesses as to how things work don't extend very well to the Boltzmann brain scenario where you start from thermal equilibrium from maximum entropy and you randomly fluctuate into a lower entropy state. That's not something that ever happens in our experience but eventually it would happen if you waited long enough and that's where the Boltzmann brain scenario starts. But then if you ask what is the most likely future of such a configuration it's exactly the time reverse of the most likely past because there is no arrow of time in this situation. So whatever situation you get into the prediction for the future is you're gonna get out of it in exactly the time reversed way. There's no preference for a complex system to stay complex. The preference is to go back to thermal equilibrium.
44
2024.0643Rue Phillips says tell us about your best France experiences. I just booked a trip for my family to go to Chamonix in the Alps and Avignon in the south. We're trying to get more out into country and nature. We've done Paris and Normandy more than once. My favorite red wine is Pinot Noir and I hear that red burgundy from France is like that. Do you have experience with burgundy or do you prefer other French wines?
Yeah I hate to sound like a broken record I guess but look people are different. People are going to have different experiences in France. Jennifer and I are not get out into the country people. We're happy to do that happy to get out into the country but that's not our primary drive when we go elsewhere to visit around. We're more city people and sometimes individual excursions like this I don't sorry I need to slow down here because when I put up the call for questions on Patreon I mentioned that I was going to France for about 10 days. I had just come back and we'd had a lot of wine and a lot of asparagus. Asparagus must have been in season. There was a it was on every menu that we went to while we were in France. So the questions reflect this and I don't know if the people who are listening who are not who had not read that Patreon post know what we're talking about here. We're talking about my recent trip to France. Anyway so we went to Bordeaux which was a lifelong dream or at least a dream that Jennifer and I had had for many years because those are our favorite kinds of wines. Bordeaux and Burgundy are probably the two biggest wine producing regions in France but there are many others Coone and the Lair Valley and so forth. Not to mention champagne. Very very roughly speaking burgundy wines are lighter and yes they are mostly pinot noir. Whereas Bordeaux Wines well there's the right bank of the left bank. The left bank is mostly going to be Cabernet and the right bank is going to be mostly Merlot but they all mix together In Bordeaux there's a list of six varietals of grapes that you're allowed to use. I don't even think Pinot noir is one of them. So you're not even allowed to use Pinot Noir in Bordeaux but we like Bordeaux's the best. The thing we learned is that and this might not be true but the impression we got from our few days in Bordeaux is that at the less expensive level at the wines you might pay 20 or $30 a bottle for. And they're going to be generally they're not gonna last as long. They're gonna be drinkable within a few years. You're not gonna wanna wait 20 years to drink them. In that category we liked the right bank wines better Saint-Emilion and regions like that but the more expensive wines we like the left bank wines better Chateau Margaux is one of our favorites actually as as I already mentioned. So Château Margaux is the one we got to visit and we visited Chateau Preshak in the right bank. And maybe our experience is absolutely not very comprehensive here. So our impressions might be wrong but we like the ways that the left bank the Médoc region wines age better than we did. The we got more out of them than the pricier right Bank wines. In terms of experiences yeah we like to visit around a little bit walk as much as we can. 'cause walking is a great thing in France in a way that the United States is just not adapted to that. It's always fun to go to museums but may is sort of the height of tourist season and they're all very crowded so we didn't do much of that. So a lot of it a lot of our time was spent other than visiting the vineyards. We took one class about Bordeaux wines which was super interesting. And then we sat around and ate and drank [laughter] not like into excess but to excess in time. Not an amount of food or drink but we would just have a good meal and sit in the cafe and watch people walk by and have a great time. That's and we didn't do any work. We didn't bring our we had we brought our laptops 'cause we aren't inseparable from our laptops but we almost didn't open them up while we were there which was a great thing to do. But France like many other countries as a whole country there's many other things to do. I have friends who visit France regularly just to ride bikes across the countryside. I mean obviously there's a lot of history there. A lot of great landscape and places you can go to the country and escape. I wouldn't be the person to ask about that but if you love it then it's a great place to do those kinds of things.
45
2024.0644Brian Mendoza says if it was discovered that the United States had been concealing contact possession or knowledge of non-human technology would you find that upsetting or justified?
Well let's be very clear. I think that the chances that's true are essentially zero. So I'm not worried about this question really. But let's put it this way. If the United States does discover such things do I think they should keep them secret or let people know? They should let people know. I think that I suspect that governments overall err on the side of keeping secrets more than they should. And certainly for something as super duper important to the nature of being a human being as that I think that they should let people know right away.
46
2024.0645Kevin James says I was listening to your solo podcast number 270 and at the point where you were talking about AGI and cyborgs it seemed to be that you were so close putting these two together to have a new worry of a super intelligent cyborg. What if this ends up being our path to super AGI would you increase your credence to this outcome occurring in the next 20 years? I suppose it's not purely artificial.
I don't think I'm worried about cyborgs. Cyborgs I take it and maybe different people have different attitudes towards it. But I think that to me a cyborg is one that still has the human mind the human brain but is adding cybernetic parts to its body in some way that might be very very important and very useful. I'm slightly of the opinion that improvements in biology are going to lead to bigger differences in how human bodies are repaired and modified in the future than in other kinds of technology. But who knows? I'm certainly very very open to different possibilities there. I just don't see that as a major phase transition unless what you mean is somehow melding silicon based thinking to more biological based thinking. I don't know whether that counts as your scenario. That's something that I'm very very open to but I know nothing about. So I have not even enough information to worry about it right now.
47
2024.0646I'm gonna group two questions together. One is from Jonathan Cart Where do you stand on the positive integers summing two minus 1/12? My understanding is that there are useful implications of this result in string theory. Is this mathematical hocus pocus or is it hinting at something interesting about the nature of infinity? And Scott Lewicki says an infinite number of mathematicians walk into the 29th Street tavern. Each beer costs a dollar. The first one pays for one beer the second one pays for two beers. The third one pays for three beers and so on. In the end the bar owner lost eight and a third cents. 112th of a dollar some particle physicists a table nearby drinking wine of course see this toast to mathematicians and decide to apply this to obscure particle physics. I am sitting at a corner table very very confused watching this and decide to switch to hard liquor. Please please explain what just happened.
So I don't know what brought on two different questions about this exact same topic. Maybe the fact that we had a string theory episode recently but there is a well-known mathematical curiosity that is sometimes explained as if you take the positive integer 1 2 3 4 and you add them together one plus two plus three plus four etcetera what answer do you get? You might say well the answer is infinity but you're told that the answer's actually minus 1/12. What in the world can that possibly mean? How in the world can you add up integer and get an answer that is not an integer? How in the world can you add up an infinite number of positive numbers and get a number of increasing positive numbers and get a number that is not infinite? How could you add a number of positive numbers and get a negative number? Like all of this makes no sense at all. So the answer is that of course when you're told the sum of the positive integers is minus 1/12 you have to define what you mean by that and no and that people are intentionally hiding from you what they mean by that. Because it's kind of sounds boring when you actually explain it but the provocative claim gets more people talking about it. So of course the sum of the positive integers stated just like that is meaningless. It's not a number right? There's no actual limit that you approach As you add up all those numbers together. When you can add you can add an infinite number of numbers together. If the sum of them approaches a limit like you can add one half plus a quarter plus an eighth one over two to the end there's an infinite number of them but they approach the limit of the sum being one and that's perfectly well defined in a unique way. That's not the case for one plus two plus three plus four etcetera. So what you do is in the back of your mind or explicitly you say look rather than adding up the positive integer I'm gonna write down a sum of numbers as a function of some other numbers. So I'm gonna add up I forget what it is you're gonna I'm gonna get it wrong. If you add up n to the power A for n goes from zero to infinity okay? But as a function of the number A right? I'm not gonna tell you what A is but just as a general function we'll leave a undetermined add up N to the power A and that would be something you could do. For example if A is minus a half [laughter] then you get the sum we just said a half plus a quarter plus etcetera. But then you can look at it for different values of A. And so you play a trick like this where you have something which for certain values of the parameter A is perfectly well-defined. And then you extend it and you take a limit and you say that okay I'm gonna approach a limit where this sum looks like one plus two plus three plus four plus five plus six etcetera. And I find that the limit of this function as it approaches that value is equal to minus 1/12. So basically what you've done is you've regularized the problem you've expanded the question what is one plus two plus three plus four to a different question? What is the sum of what is the function defined by the sum of these numbers? And then you take a limit in a certain way to get the answer. It's minus 1/12. And then you proclaim that that is actually the answer to the question that you started with. Is it? Well it is with all the additional caveats that that's what you really meant by that. It turns out that this particular question in the string theory is very well defined. You're calculating the vacuum energy and string theory on the string world sheet itself. This kind of mathematical expression shows up. So I don't think it's anything very deep. It's it is kind of both at the same time a trick and something that is true in the particular case where you care about it in string theory. It's certainly not anything to worry about too much. No one is pulling your the wool over your eyes nor is there something deep about the nature of infinity that is hard to understand. Neither one of those is the case.
48
2024.0647Nevada city Bob says as I understand it during hawking radiation one particle with positive energy escapes from the black hole while another nearly identical particle with negative energy remains inside. Does negative energy exist anywhere other than black holes?
Well again this is an example much like the previous one where people are telling you somewhat sensational things because it sounds more impressive than the reality. What do you mean by energy in this case? The thing about being inside a black hole remember is that you can't be stationary inside a black hole from the perspective of someone who's outside the black hole. Because you're separated by an event horizon from the perspective of the people inside the black hole you'd have to be moving faster than the speed of light to be outside the black hole. So what is actually going on is that you can define a quantity which has the interpretation of the energy of a particle as it would be measured by an observer outside the black hole indeed infinitely far away from the black hole. If they could do that it's a slightly weird quantity defined but mathematically you can do it right. What would the what is the energy of the particle as seen by an observer at infinity? Okay. And that actually is a meaningful thing because if you think about it in relativity relativity the word relativity means that things like velocities are only defined relative to something else. And the energy of a particle certainly depend on it depends on its velocity right? The kinetic energy depends on its velocity. So when you're inside a black hole versus outside that's a tricky thing to define it. There is a way to do it but what it leads to is that there can be particles which from the perspective of someone at infinitely far away from the black hole the energy of a particle of a certain kind of particle inside the black hole can be negative. Now they're infinitely far away right? It doesn't matter. You can't make a negative energy particle right in front of you. If you were falling into the black hole and you asked about the energy of the two particles making up the hawking radiation one going in and one going out they would both have positive energy but that doesn't matter to the point of view of someone who is infinitely far away. They're the ones the infinitely far observer who are going to measure how the black hole as a whole is changing in its energy. And the answer is it's going down in its energy because it's emitting radiation to the outside. So it sort of has to be it has to be absorbing particles of negative energy from the perspective of the outside observer.
49
2024.0648Chris Gunter says would you use a Star Trek teleporter one that disassembles you on one side and reassembles you on the other? Let's assume it's as reliable as in the show but what if you effectively die when going in and a different consciousness pops out on the other side?
I don't think that's true that you do die when you go in. I know people have said this but this is another classic example of where our intuition doesn't work because our intuition is not trained on teleporter machines. We don't have any such things transporter machines. They would actually be called in Star Trek. So because we're actually trained in situations where there is physical continuity over time in most of the stuff that makes up our bodies individual atoms come and go inside our bodies right? But it takes a while there. Overall the bones and the organs in our bodies are more or less persistent over time. So we associate that with something real something fundamental. There is you you are a person you have a body there you are etcetera etcetera. If you're more strict about it if you're more careful then the you that exists at one moment of time is not the same you that exists a minute later even in the regular old world. Forget about transporter machines etcetera okay? You're in a slightly different configuration of stuff. You're in a slightly different position in space and so forth but there are natural obvious reasons why it makes sense to associate that as one single person evolving over time rather than talking separately about you at 1:00 PM you at 1:01 you at 1:02 etcetera. The physical continuity there, goes hand in hand with some informational continuity. So not only are they the same molecules in your body but they're in that more or less similar pattern right? The pattern is slightly changed over time but it persists. And what happens in the transporter machine is that the pattern reappears but with different atoms somewhere else. That is not something that we ever deal with in the real world. But to me what matters is the pattern not the actual atoms that you're made out of. So to me you're just still alive. I have no trouble whatsoever stepping into a transporter machine if it is perfectly reliable because I think I would call the me that came out the other side exactly the same me as the one that went in in precisely the same sense that the me at 1 o'clock is the same as me at 2 o'clock.
50
2024.0649Mark Kumeri says I was under the impression that cosmic inflation was generally well accepted among cosmologists and theoretical physicists. Recently it seems that several prominent physicists have expressed their doubts with inflation. Listening to one of your podcasts it seems like you are not entirely convinced as well as I believe you think about a theory might come along even if inflation might be the best one today. Can you elaborate on your concerns with inflation and perhaps discuss the viability of the inflationary multiverse?
This is a very long conversation. I would encourage you to read my book from Eternity to here where I talk exactly about this. I think that inflation is a great theory and it may or may not be right at the most basic level. The energy scales and the physical processes that we talk about during inflation are very very very far away from anything we've directly probed experimentally. So in that sense it is entirely speculative which is fine. Speculation is fine but we shouldn't get too attached to our speculations. And the other big worry is of course that I think that we don't have a good theory of how and why inflation started. People cheat about that all the time. They think that if it's simple it can't be too difficult for it to start. But it's very very low entropy the condition you need to make inflation go. And as Roger Penrose has been emphasizing for many many years you need to explain why a physical situation is so low entropy. 'cause low entropy means there's a very very very tiny fraction of all the possible micro states you could have been in that you actually are in. Someone like David Albert philosopher of science will just say well that's a law of nature. There's a law of nature that you started in this kind of low entropy state. I think most physicists would want an explanation of that and we don't have one. And then finally the multiverse of course as we just discussed in an earlier question has these raises these problems with predictability and being able to understand why certain things are more likely than others as Paul Steinhardt and others would argue. So I think that there's plenty of reasons to worry that inflation is not the final answer despite the fact that it's a very good theory. And I think that in that situation you have to keep an open mind. I think that's fine to do but you're right a lot of working cosmologists just take it for granted that inflation is right. I think that they're a little bit premature in doing that.
51
2024.0650Humberto Nani says so how did Ariel manage to get the Quanta and Fields book?
For those of you who have bought Quanta and Fields the dedication is to Ariel. Book one was dedicated to Jennifer. We have a three person household here. Sorry. Well, me plus three others, Jennifer, Ariel, and Caliban. So Caliban will get the next one. He will get complexity and emergence, which will be volume three. Given that Jennifer got the first one, which I think makes sense, the question was between Ariel and Caliban, who should get the quantum mechanics book and who should get the complexity and emergence book. Caliban is just more of a chaos agent than Ariel is, whereas Ariel is a little delicate and unpredictable. So I thought that Ariel fit better with quantum mechanics, Caliban fit better with complexity. But honestly, they could have gone either way. Now, of course, we have a stray cat puck, who hangs around outside. Maybe I should write a fourth volume just to give him something to dedicate to.
52
2024.0651Alex says we all remember pictures of particle tracks in bubble chambers. They allow us to determine the momentum and location at the same time. It looks like a contradiction to the Heisenberg uncertainty principle. What am I missing?
You're missing numbers. So, look, I've seen pictures of cars moving through time, right? That doesn't violate the uncertainty principle just because the uncertainty is small. And I think the same thing is going on with the tracks in the bubble chamber. If you actually run through the numbers and look at the precision with which you are measuring the location of those particles, it is not infinite. In fact, it's a little crude, right? And if you try to improve the precision with which you are measuring the locations of those particles, you would find that their momenta become disturbed by those measurements in 100% compatibility with the uncertainty principle. So people would have noticed if that was actually a contradiction. Don't worry.
53
2024.0652David Maxwell says, it seems to me there's a tension between a kill switch to prevent AI getting out of control and the right to life of a conscious AI, when the latter is potentially a key argument for the former. But how will we tell if an AI develops consciousness, given its subjectivity?
Yeah, I think it's a good question. I literally have no good things to offer here. I kind of think that we've done a crappy job, both within science and philosophy, of taking seriously this question of when will AI really count as conscious? It's nowhere close, in my opinion right now, as I've said in other places, we have developed these large language models that are incredibly good at mimicking consciousness without actually getting it. But that's a different thing. We should take that difference seriously. There's a great thread on Twitter where I forget who did it. I would like to give them credit, but they talk about a professor who did the following thing. He took a pencil. And he attached two googly eyes to the pencil and then he held it up to his class and he said, hi, I am Tim, the helpful pencil. I will help you think and write and express yourself in useful ways. I'm really glad that we're going to have a relationship where we both work together better. And then the professor suddenly snaps the pencil in two and the whole class goes, they're shocked. How could you kill Tim like that? The helpful pencil, of course, it's just a pencil, right? But the professor then says, this is why there's a lot of hype about AI. And that's exactly right. AI, again, as I've said many times, is going to be hugely transformative in many ways. That doesn't mean that it's actually a conscious agent. What it means is that they figured out a way to make it act like a conscious agent. And we human beings, again, because of all of the training that we have had in dealing with the world before computers came along, is that things that act conscious are conscious. Now we have something different, things that act conscious without being conscious, and we don't know how to deal with it. We assign, as once again Dan Dennett explained to us years ago, we assign agency, we assign intentionality to things that seem to exhibit it. But we have to think a little bit more carefully about whether that assignment is correct or worthwhile in a practical sense, in regimes where we have not yet been experienced to previously in human history.
54
2024.0653Michael Gibbs says, I have recently seen several articles claiming evidence has been found that dark energy is weakening over time and how this is rocking the foundations of physics. To me, this falls into category of extraordinary claims require extraordinary evidence. I'm curious if you think the data is strong enough to take this seriously, and if so, what are the major implications? I mean, the data are strong enough to make you be aware of the possibility. The data are not nearly strong enough to make you think it's true with some high level of confidence.
What Michael's referring to is various claims, especially from an instrument called the dark energy spectroscopic instrument, D-E-S-I. DESI. There's too many things called DESI in physics, sadly. So you have to spell it out every time. But they have a new result that says, without a lot of statistical significance, that their data is most compatible with the dark energy slowly decaying over time. I'm not going to get too excited about it. These are very, very difficult observations to make. The dark energy could decay over time. That's absolutely possible. I think that having it be constant overtime is much more robust and plausible. So I'm going to bet that I'm going to put most of my credence on that until the evidence becomes a lot stronger than it does, than it is right now.
55
2024.0654Joel Rembach says, if we can't access any of the many worlds, what is the purpose of studying it when there are still many discoveries to be investigated in the reality we exist in?
Look, I've said before, I don't care about the other worlds. I care about the laws of physics. The question is, we have problems with quantum mechanics as it is taught. It is not a sensible, rigorous, coherent theory. We make things up like observations and wave function collapse that aren't rigorously defined. Many worlds is a well defined theory that replaces the ill defined Copenhagen interpretation and lets us actually do science with it. The point of many worlds is not the other worlds. It's that we've answered the question of what is a measurement? Why do you get probabilities and things like that? And equally importantly, there's the fact that we don't know the fundamental laws of physics. We're not done with physics yet. We're trying to build better laws of physics. And I strongly think that taking quantum mechanics seriously and thinking about what is the correct foundational version of quantum mechanics will be useful to that program. So again, it's not about the worlds. People who really obsess about the other worlds are the ones who haven't really internalized many worlds. Many worlds is just, it's always obeying the Schrodinger equation. That's the essence of the theory. G Agnes... I'm sorry, I should have practiced ahead of time.
56
2024.0655Agonacian says, listening to the podcast with Claudia Durham about how gravity could be an emergent force and your own explanations, how the macroscopic reality we observe is emergent from underlying elementary physics, I have a naive question. Is it possible that we may be missing a force similar to gravity, that is emergent and acts in a much bigger scale than gravity, say at architectural scale of galaxies and even galaxy cluster levels, not trying to contradict Einstein?
It's possible. Many things are possible. But of course we've thought about that. We've thought about both modifying gravity and adding new forces in so far, the theories that we have are certainly not better than general relativity plus dark matter. They're usually worse in some way or another, but there's always going to be a limit where they become indistinguishable. So there's no evidence for any kind of thing like that. And there's no obvious benefit to doing it. So it's not a very popular way of thinking right now, but it is possible. And so we still keep contemplating these other theories because unless you write down what the theory is and think about what it predicts, you don't know how to test it. Right. So that's what we're trying to do as modern cosmologists right now.
57
2024.0656Marc Beranger says, I hope you enjoyed your trip to France. What other places in the world or off-world would you like to visit and why?
I'm going to erase off-world from that. That's a whole other category that I haven't really thought about and we don't have the capacity to do. But in the world, I've traveled a lot, but not a lot a lot. Like, I'm not a super duper world traveler, like some of my academic colleagues are. Never been to India, never been to North Africa. I would love to go to, like the north, the Maghreb region, right. Tunisia, Morocco, Algeria, those kinds of places. I've been to Spain, but only very, very quickly and not really very seriously. I've not been to eastern Europe, really. I've never been east of Germany, within Europe. So I have done a lot of western Europe, South America, different places in Asia. And it's always a temptation. Like, we had so much fun when we went to Hong Kong, and we had so much fun when we went to Japan, and we were very, very briefly in Singapore, and that was great. And Vietnam, all those places in Southeast Asia we would love to go back to them because we didn't spend enough time. But then again, I've never been to Marrakesh or to New Delhi. So, like, do we go to someplace new? I don't know. I would love to do all these things. It's a big world out there. I don't think I'm ever going to run out of places I want to go.
58
2024.0657Tomer Hakowen says, are there examples of math notation you find particularly nice or elegant? What's the most commonly used notation, in your opinion? Sorry? What's the worst commonly used notation, in your opinion?
The worst notation, actually, I'm sorry, I didn't really read this question carefully. I only read the first half of it. So the first half of it, I think my answer is index notation in differential geometry and relativity. In fact, I had this joke back from graduate school that the reason why notation was so much more beautiful in general relativity than in quantum mechanics or quantum field theory is because in quantum mechanics and quantum field theory, the theories have just been so useful and important ever since they were invented that people just put them to work right away and spent time modifying the theories and changing them, whereas the pace of change and progress in relativity is much slower. So people had the time to come up with really good notation. Index notation is a way of dealing with the fact that there are these higher order tensors, like the Riemann tensor and so forth. There's versions of it in field theory, but they're not very beautiful. And different people use different notations and things like that. I don't know. I think that mathematicians like different fonts, they like different kinds of notation, and it's a little off putting to the outsiders, and I'm not deep enough into it to be very well versed in different kinds of mathematical notation. So I'm just going to stick to my own wheelhouse and say, index notation in differential geometry, the worst notation, I don't really know. I don't have any, I'm not going to. I haven't dwelled on that. So I'm sure if I went through all the notations I can think of, I would be able to come up with something. But nothing pops into my head right now.
59
2024.0658Captain Brick says, I've thoroughly enjoyed your book, Something Deeply Hidden. One concept I've grappled with is the self-locating uncertainty and the thickness of the branch. I think I've conceptualized the thickness of the branch as the number of copies of each possible outcome. For instance, if an observed event has two possible outcomes, A and B, with probabilities 0.3 and 0.7, the world splits into 100 branches, 30 with outcome A and 70 with outcome B. And then the question goes on a little bit. Is this a valid way of thinking about the thickness of the branch, or am I way off?
Well, I would say that it's halfway valid. Let's put it that way. In quantum mechanics, if you have two branches of the wave function of the universe, one with probability 0.3, the other with probability 0.7, you have two branches of the wave function of the universe. You don't have 100, right? So the statement the world splits into 100 branches, that's an empirical question. Does it or doesn't it? What you can do is say if I split the world into branches with the proviso that all branches needed to have equal amplitude. Okay, let's ask that question. Could I split the world further? It's now two branches, one with probability 0.3, one with probability 0.7. I want to keep splitting it so they all have equal, well I shouldn't, you shouldn't call them probabilities. If I'm being a careful philosopher here, what you mean is. Weight. Weight is given by the amplitude squared of the wave function. And the question we're trying to ask is, is it okay to identify the weight with the probability? And, well, if you want to ask if I were to keep branching the universe until I had equal weights on all the branches, then it would be true that the relative number of remaining branches is given by the weight. So if you start with prob... With weight 0.3 and weight 0.7 and kept branching until you had equal numbers of branches, sorry, equal weights to all the branches, then the ratio of branches of type A to branches of type B would be 0.3 divided by 0.7, and that would be the probability also. So that would be, that would be perfectly sensible. But the reason why that's not an automatically obvious thing to do is that it just doesn't necessarily happen, that the universe does, as a matter of fact, branch into branches with equal weights. The skeptic would say that sort of is begging the question, right. Why should I care about branches with equal weights? What is so special about those? Once you make the statement that branches with equal weights are special, in particular, they deserve to be given equal probability. Then you get The Born Rule, then the probability just comes right out. That's the big step. The math is not the hard part. In fact, if you say when and only when I have branches with equal weight, they give equal probability. That seems like a very natural thing to say. Then you get The Born Rule, then you get the whole thing. You get the probability is the wave function squared. That is the only extension, that's the only generalization of the statement that branches with equal weights get equal probabilities. But that's a contentious statement. Some people are going to buy that, and some people are not. I don't know what to say.
60
2024.0659Paul Hess says, growing up in the seventies, it seemed we were evolving out of an emphasis on the melting pot and toward an emphasis on individual cultural pride, which seems like a good and noble thing. Over the decades, that emphasis kept increasing to the point where melting pot type perspectives were even looked down upon. Has this pendulum swung too far? Do you think this led to the increased divisiveness within our society? Do you agree with my observation that the emphasis shifted in the first place.
Well, I don't know. I mean, when you say things like there was an emphasis where? Who had that emphasis? I think that we live in a very big country, and this is a very US centric question. I think different people had different emphases. Okay. I think something that I would get behind is the idea that in the '70s, maybe starting in the '60s, but certainly in the '70s and '80s, an increasing number of people realized that the melting pot way of thinking, even though it kind of sounded virtuous and good, we're all in this together, et cetera, et cetera. All of that kind of rhetoric where, let's all get together, we're all virtuous and good. Let's all be one unified country. That tends to end up, as a practical matter, favoring the people who are in the better position already, the dominant sectors of society and squelching or ignoring the people who are in smaller, less powerful sectors of society. So there was a countervailing current. And so here I'm agreeing with you. That said, I wouldn't put it about cultural pride. I don't think that's a big deal. But you can respect differences between people while giving equal dignity to all different kinds of people. You can be different, yet deserving equal dignity. I think that's the point. That's the best way of putting it. Has the pendulum swung too far? You know, like, I'm not a big believer in individual cultural pride. I think that it's fine if it's completely frivolous, like, look, I'm a sports fan. I root for the Philadelphia 76ers for completely non-rational reasons, not irrational reasons. It's not like bad to root for the Philadelphia 76ers, but I only root for them because I grew up in suburbs of Philadelphia and they were really good when I was growing up and I became a fan. I am not at all a fan of the Boston Celtics or the Los Angeles Lakers, but maybe if I grew up in those regions, I would be. I think that that's just harmless, right? That's fun. You know, you can root for your team. I mean, it's not super fun to be a Sixers fan, I have to admit, right now, but in principle it could be harmless fun. But to take too much pride in what country you were Born in or whatever, I think that to the extent that that helps motivate equality and fairness overall, it's good to the extent that it puts other people down. It's bad. And so I think you have to balance it a little bit.
61
2024.0660Rad Antonov says, could you please clarify what Vafa was getting at with his argument about black hole mass versus charge in Planck units?
Yeah. This is something called the weak gravity conjecture. And the idea is supposed to be a conjecture came from Nima, Arkani-Hamed and others originally that gravity is the weakest of all the forces. So that sounds like a sensible conjecture. Gravity is pretty weak, right? It seems to be weaker than electromagnetism or the strong nuclear force, or even the weak nuclear force, for that matter. But here's the problem with gravity as the weakest force. If you calculate the force between two neutrons, neutrons have gravity because they have mass, but they don't have any electromagnetic force between them. So in what sense? Because they're neutral. So in what sense is gravity between two neutrons weaker than the gravity, than the electromagnetic force between two neutrons? So what you have to do is define very carefully what you mean by the claim that the gravity is the weakest force. And of course, you look around at electromagnetism, and similar arguments will work for the nuclear forces, but it's more complicated. So forget About those, you notice that when particles have a non zero electric charge, there's a minimum value, right? One, actually no, a third for, because quarks have, can have plus or minus one third electric charge. But anyway, there's a minimum value. And so you need to define what you mean by gravity being the weakest force, as the gravitational force between charged particles, always being weaker than the electromagnetic force between charged particles. So there's a way to turn that into a statement about black hole masses versus charges. But that's basically the important thing that he was trying to get at, the idea that gravity is the weakest force.
62
2024.0661Eric Coker says, I'm loving the new book. It's like I have the text to go along with the class now. I'm still struggling with the whole modes to number of particles thing. In QFT, a specific mode gives you energy levels, and the amplitude of the mode gives the number of particles to that energy. But what about location? Specifically, on page 94, you show how to get a wave packet out of a combination of modes. What leads to the tying down of that combination of modes in QFT? The guitar string analogy makes sense for the wave function of a particle, but these modes of field seem like they should extend out forever, and so any combination of them would do the same, right.
So this is clearly a failure on my part, because I did try to explain this in the book, but apparently it didn't come across, and that makes perfect sense because it is very abstract. So when you have the wave function of a particle, you normally, more often than not, even though you don't have to think about it as a function of position, right? Psi of x, and then Psi of x goes to zero at plus or minus infinity, because the wave function is normalized, right. The integral of the square of the wave function equals one. So you can't have infinitely much wave function. You only have a finite amount of wave function. So it has to go to zero at infinity. When we go to quantum field theory, we start thinking about the wave function of a mode. And a mode means a certain profile of the field with a fixed wavelength stretching forever throughout all of space. The only thing that, in this slight simplification, the only thing that matters about the mode, is how is its amplitude? I hesitate to use the word amplitude because it doesn't mean the quantum amplitude, it just means the height of the wave. And so I call it the height in the book. So we have the wavelength that's fixed, but the height of the wave is something that we can think about. So the wave function of the field is a wave function of the height of the wave. It does not have a dependence on where you are in space. Okay, so the tying down that is relevant is that the wave function of the height has to go to zero as the height goes to infinity. Does that make sense? You have Psi of the height of the mode, and you have that separately for each different kinds of modes. And so through the magic of Fourier transforms, you can sum up modes in such a way that you have a wave packet that has a relatively localized profile in space. So it's very abstract. There's a lot of steps to get there. The math all works out. But I'm sympathetic if it doesn't make perfect sense the first time through.
63
2024.0662Ben P. Stein says in the string Theory podcast and his recent review paper, Professor Vafa seems to say that almost all naively consistent, effective field theories would be ruled out in a quantum theory of gravity. Is the effective quantum field theory explained in your new book an example of a naively consistent one that is ruled out?
We have no idea. I mean, we hope not. String theorists hope not. So the idea here of the swamp land is that, as Ben just said, most effective theories that you could write down are actually not consistent in a world with gravity, there is no ultra violet completion of them. So they are in the swampland, they are not in the landscape. But we don't have a very detailed view of which ones are ruled out and which ones are not. So if the standard model of particle physics plus gravity is in the swampland and is ruled out, is not consistent, that would mean that string theory can't be right, because that's the theory that works well in our world. But we don't know. That's something that we're trying to figure out.
64
2024.0663Jonathan Goodson says a question about information loss, when the Schrodinger wave collapses and a specific outcome is observed, is the information that was contained in the wave lost, or are the specifics of the wave not considered information?
Yes, information is lost. And I do apologize. There was another question, maybe more than one question. I didn't get around to this time, about different notions of information. So, yeah, there are different notions of information. In the context of the phrase information loss. In quantum mechanics or quantum gravity, the information we're talking about is the information contained in the wave function, the information needed to completely specify the quantum wave function and measurement that collapses the wave function destroys that information. One way of saying this is if I give you a wave function and say, I'm going to measure it, what is the probability of getting different measurement outcomes. I can tell you that, right? I can just square the wave function. I can tell you the probabilities. If I instead say, I got a measurement outcome, the spin was up. What was the wave function before I measured it? You have almost no idea, right? It could have been many, many different wave functions that could have given you that measurement outcome. So information has been lost. There's no going backwards in the conventional view of quantum mechanics.
65
2024.0664Brian says, I love teaching. I have a bunch of degrees, but I don't care about teaching. Should I? And why do you like teaching, especially for general audiences? Oh, I'm sorry. I got very confused by Brian's questions there. His question started, I love learning, not I love teaching. So he says, I love learning. I have a bunch of degrees, but I don't care about teaching. Should I? And why do you like teaching, especially for general audiences?
No, I don't think you should. Some people like it, some people don't. Some people are willing to do it because it's what gets them a job. Some people do it well. Some people don't do it well. Some people just devoted to doing research or doing learning or whatever. Look, some people don't even like research. They just like learning the things that other people have already figured out. You know, research is defined as figuring out something brand new. Some people would be happiest if they could get paid just to learn the physics that other people had discovered. Sadly, the world does not work itself out so that we can get paid to do whatever it is we most like, right? There's some optimization problem. We each have to go through balancing what we actually want to do versus what the world will let us get away with doing. So very, very often. If what you want to do is academic research, your job is also going to involve teaching. I like it for lots of reasons. I like it because. Not necessarily in order, but it improves my understanding. Right. When I really think about things and try to explain them, I end up understanding those things better. It's a reason to learn new things. Sometimes I teach classes where I only know a bit about the subject and so I can force myself to learn by teaching it. I get joy out of other people understanding things. That's why I have a podcast. That's why I write books. That's why I teach and, and give talks and things like that. It improves people's lives. People like it. People appreciate it. I constantly get, constantly is an exaggeration, but I frequently get people saying, yeah, I read your book years ago, and that made me become a physicist. Or that just gave me happiness at a time when it was very useful to do that. And so I personally get a lot of value out of it. If you don't, that is not a moral failure on your part. There's plenty of things that other people value and get value from that I don't. That's okay. Lots of different people out there in the world.
66
2024.0665Catherine Traub says, when you're eating a spear of asparagus. See, I told you. I mentioned asparagus in the call for questions. When you're eating a spear of asparagus or drinking a glass of wine, are you thinking about physics?
Sometimes I am. Sometimes I'm thinking about physics, sometimes I'm not. One of the beautiful things about theoretical physics, or philosophy for that matter, is you can think about them at any time. So it is absolutely true that sometimes when I'm having dinner or walking down the street or whatever, I'm deeply thinking about problems in physics or philosophy. Often not. If I'm having a conversation with somebody that is not about physics. I am not thinking about physics while I am doing that.
67
2024.0666Tim Converse says, with regard to the arrow of time in the past hypothesis, to what extent was the early universe lower entropy than the present simply because it was denser? Can we imagine alternative early universe configurations that would have been equally dense with much higher entropy?
Yes, very, very easily. The early universe was smooth, it was homogeneous. And because gravity was so strong, a less smooth configuration would generically have had higher entropy. A universe with a lot more black holes at that early time would have had a lot higher entropy. So it is very easy to have much higher entropy states than we had in the early universe. So it's not just about the density, it's much, much more about the smoothness.
68
2024.0667Dan Cohen says, I love the biggest ideas in the universe. Books not done with the second one yet, but so far so good. But I really wish I could follow the math better. Are there any online courses you think are good foundations or not too text booky books? For example, in the first book, once things got to tensors, it got hard for me to follow, and it even took me a couple tries just to figure out that linear algebra was the math topic I was looking for.
Short answer is no, I do not know specific good online courses, et cetera, but they're there. I know that. I know there are online courses. And also look for lecture notes, like, for whatever reason, and I've done this myself, but scientists and mathematicians love to take time out of their busy day and write up pedagogical lecture notes that you can find online about different topics that you're interested in. So very often you can find something like that. I don't know, because I did the whole physics education, undergrad, grad school postdoc thing, so that's how I learned it, and that was many years ago. So even the resources that were best back then are not the best ones now. So I don't know which ones are there, but I'm sure they're there, and I encourage you to look for it.
69
2024.0668Cooper says. Is there an intuitive explanation for why the SU-3 group being non-abelian leads to gluons carrying the color charge? Or is it simply a fact that comes out of the math and doesn't lend itself to analogy or simplification? Explanations that I've seen have jumped between these two facts without explaining the connection.
I suspect the answer is that I don't know of a really good intuitive explanation for that. I can say words that are true, but whether they add up to something that you qualify as a helpful explanation, I can't promise you that. I mean, when a group is non-abelian, that means that there's some symmetry transformation, like, you're rotating a sphere or whatever with the property that it matters when you do two different operations, which order you do them in. So if you think about a circle and I have rotations of the circle, so I could rotate it clockwise by 20 degrees and then counter clockwise by five degrees. Right? So the overall change is clockwise rotation by 15 degrees, 20 minus five. It didn't matter whether I did the 20 degrees clockwise first and then the five degrees counter clockwise or the other order. They go, and they add up the same way either way. So that is an abelian transformation. Whereas if I have a sphere, so, like a two dimensional sphere embedded in three dimensions, and I rotate it by 90 degrees around the X-axis, and then by 90 degrees around the Y-axis, I end up with the sphere in a different configuration than if I did the Z-axis first and then the X-axis or the Y-axis first, or whatever it is, if I change the order of the operations. So that is a non-abelian group. And very roughly, the gauge symmetries based on non abelian... Well, this is true. Gauge symmetries based on non-abelian groups have gauge bosons which interact with each other. You can say that they carry the color charge, that there's a truth behind that. But I think the relevant thing is they interact directly with each other. Gluons, W-bosons, Z-bosons, and photons, which are based on an abelian gauge group, do not interact directly with each other. They interact indirectly because they interact with charged fields like electrons and positrons, and then they interact through virtual particles. But there's no direct interaction, photons to photons. So, roughly, the difference is that the gauge bosons... I mean, the gauge bosons, can be thought of as carrying with them knowledge about an infinitesimally tiny rotation, an infinitesimally tiny symmetry transformation. The technical terminology would be the Lie algebra of the Lie group. That's a completely useless bit of nomenclature for you if you're not into it already. But just so you know, if you ever hear it, an infinitesimally tiny symmetry transformation is the algebra associated with the group. And likewise, if the group is non-abelian, the algebra is non-abelian, et cetera, et cetera. That goes hand in hand. And so you can think of different kinds of gauge bosons as individually representing different infinitesimal symmetry transformations. And then the leap where you're not going to be happy with this, but the leap is if these different kind of symmetry transformations, if it matters what order you do them in, then at the level of the particles that arise under quantization of this, that means the particles bump into each other. That means the particles can interact with each other. I could go into a little bit more specificity if you were happy with thinking about the Lagrangian of the fields, the kind of Lagrangian s on which you base the equations of motion for these guys involve, basically boson one times boson two, minus boson two minus boson, times boson one. And the minus sign is important there, because you're trying to maintain gauge invariance. And if the symmetry is abelian, then boson one times boson two, minus boson two times boson one equals zero. There's nothing there. But if they're non-abelian, then it doesn't equal zero. And so that shows up as an interaction between the particles that is wildly unhelpful as an attempt at an intuitive explanation, but it's the best I can do. Sorry.
70
2024.0669Nate Heller says, what are your thoughts about organized labor in academic research, specifically at the graduate level? I asked because our graduate union is currently on strike to obtain a fair contract.
I'm super in favor of it. You know, I think that I understand that academia thinks of itself as special, and it's a calling. It's not a mere job. But guess what? Most graduate students aren't going to continue in that calling. They're not going to get a tenured faculty job someday. It is in addition to being a calling, it is also a job. And I think that graduate students deserve the protections that a union can provide. So I'm all in favor of it.
71
2024.0670Okay, I'm going to group two questions together. Murray Cantor says, I understand solutions to the Schrodinger wave equations are quantized even though time and space are continuous. That said, please explain why you seem to be insistent on spacetime and spacetime being continuous. Since removing infinities entails ignoring values below the Planck length, the continuum is probably a powerful approximation to a lattice spacetime. And then Lee Vermillion says, how confident are you that there is actually infinity in physics, whether that's whether this universe is infinitely large or that there are infinitely many worlds.
I am not sure where this impression came from that I have any confidence or any insistence that the world is infinite or smooth or continuous. Quantum mechanics is smooth, right? The way that we set up the mathematical formalism for quantum mechanics absolutely centrally involves smooth time evolution. There you go. But quantum mechanics might not be right. That's completely. Okay. So what I will say when I'm trying to explain quantum mechanics is that a continuum is invoked by the theory, and therefore we're going to need to deal with that. But maybe nature is different. Maybe quantum mechanics is an approximation. That's completely fine. I have no, in fact, I literally wrote a paper pointing out that you can slightly quantify, sorry, slightly modify quantum mechanics to make it completely finite, to make it non-continuous. It's kind of a trivial observation once you start thinking about it, but it's a lot of people sort of start with something like a lattice and then put quantum mechanics on top of it and they gloss over the fact that now you've made it continuous because there's a parameter in the Schrodinger equation called T, which is time, and that's a smooth variable. So it's a little bit more work to make a version of quantum mechanics that is truly discrete and it is a modification of quantum mechanics. It's not really the same thing. Also, parenthetically, the continuum is probably not a lattice spacetime, probably a powerful approximation of lattice spacetime in the sense that spacetime is probably not a lattice in any naive sense. We know that because of non-localities in quantum gravity Lattices are overly simplistic. They're overly cheap, overly local, they break Lorentz invariance, they only allow for generally local interactions, not non local ones. Once you have things like black holes and holography and stuff like that, the idea of a simple spacetime underlying... Lattice underlying spacetime is seen to be overly naïve We don't know what the right structure is because we don't know what quantum gravity is. But we're going to need to think, it would be, we would have thought of it already if it was just a lattice underlying spacetime.
72
2024.0671Roland Weber says, can you please tell us a bit more about the Santa Fe Institute? How many people are there permanently or fractal? Is it a place where everyone knows each other, or is it too big for that? Are the folks there all scientists from different fields, or also people of other professions?
Yeah, I guess it's medium sized. I don't actually know the numbers. The number of people who are there permanently is relatively small. So the number of people who are always there, not literally always, always employed by there, even if they might be visiting someplace else. How many of them are there? In the back of my mind, I feel like it's about a dozen, but maybe it's as high as 20. And I'm just not counting everybody. I don't know. But like many other successful research institutes, it's mostly a big, empty building, which they fill with visitors, either postdocs or graduate students, or simply programs that are going on where people in some area of common interest come and spend time there and talk to each other. So when you visit, the great thing about visiting is you always meet different people. You know, it's a different collection of people who are there all the time. And one of the amazing features of SFI is that they have been very, very good at picking the right people to have them come visit. And by the right people, I mean ones who get it, who get the spirit of the place, ones who are willing to talk outside their field and to think big and to ask slightly crazy questions, but in a rigorous, ultimately grounded way. That's a tough little dividing line to hit very carefully. And I think that SFI does it very well. So it's mostly scientists, but there are people, there are some writers, some artists, who pass through in various capacities just because of the nature of the people who are the majority of people there. They're also interested in those other areas. So the last workshop I was at, where Dan Dennett was also there in the investigating reality workshop, we had a talk from a modern composer, a classical music composer who played one of his pieces on the piano, and it contributed to the discussion. They're all in favor of that kind of thing, as well as, I think at least two different fiction writers gave talks at that workshop. So that's the kind of place that it is.
73
2024.0672Mikhail Maliki says, can you speculate about what kinds of physics and fundamental ontology of the world would be developed by intelligence species that never evolved eyesight?
Well, I think it would be exactly the same as the kind that we developed. You know, we develop an ontology of the world that involves things like quantum fields and neutrinos and gravitons, even though we don't see those or experience them directly with our senses, not to mention Higgs bosons and Top Quarks and so forth. The ontology of the world is highly constrained. You can discover it with all sorts of different possible sensory modalities. But I think there's an underlying factness, facticity about what it is. The path to discovering it might be very, very different, but I think that you would ultimately discover the same thing. I could be wrong about that, but I think blind people, even here on earth, I could imagine a bunch of scientists, all of whom were blind. I think they would discover physics just like we did.
74
2024.0673Dan O'Neil says, why are there so many exponents in the laws of physics, values squared, cubed, etcetera?
That's a good sounding question, but I think it might be too general to have a simple answer there. Different examples of exponents come about for different reasons. A very fundamental one is that spacetime is three dimensional, right? So when you have things like Newton's inverse square law of gravity, that's squared and one over distance squared, that comes about because the area of a sphere centered around a point goes as the square of the radius. That's a geometric reason why that exponent appears. Whereas if you look at the kinetic energy, one half MV squared, that's because there's sort of a power series expansion where it's M, so just MC squared. In the rest, mass energy plus one half MV squared plus then higher powers of V are the right ways that things appear. So that has nothing to do with the dimensionality of spacetime. That would be exactly the same even if space had a different dimensionality. So I think the sad, deflationary answer to the question is for different reasons, depending on what law you're talking about.
75
2024.0674Paul Conti says, just out of curiosity, including family and friends, students, colleagues, etcetera, approximately how many messages, posts and emails you receive every day? And do you usually manage to read them all?
I do manage to read all my... Well, that's not true. I was going to say I usually manage to read all my emails. I usually manage to read the ones that I don't already know, I don't have to read. Like when Amazon sends me an email saying your package has been shipped, I'm just going to archive that. I'm not going to read that one in any detail. Okay. But the number I get, I think. I don't know the exact number. Of course it's going to vary by a lot. And also I have a lot of filters, certain... I have a whole bunch of crackpots who send me multiple emails every day for years now, and they get filed right away. I'm not going to, I just delete them. I'm not going to go through all those. So if you just take the ones that I actually read, it's maybe 100 emails a day of that order of magnitude, which is just to say it's between 10 and 1000. But 100 might be a reasonable approximation and many of them I can just file away and ignore. It's still far too many. I can't actually answer them, even the ones that I really should or would like to with any care. It's a broken system. I got to figure out how to do better at that.
76
2024.0675Martin Leitner says, regarding your recent appearance on the closer to Truth channel, can you tell me how much thought and effort went into setting up the background? Was there professional advice involved or did you do that yourself?
No. Well, I mean, there's no professional advice involved, I think. I'm not remembering this exactly right, but I'm pretty sure. So I did a video for closer to truth with Robert Kuhn, one of the various videos I did for, when the new book came out. And I did it in the same studio that I am sitting in right now. Studio in quotation marks, just a room in my house where I record the podcasts and otherwise it is used as storage for books and music equipment and things like that. So no, there's been zero professional thought into the background. It's the same background that people who are Patreon supporters have seen in any other video I ever put up on Patreon. So I send along my appreciation for the fact that you might have thought there was any professional advice involved.
77
2024.0676Kaushik Mitra says, since my early 20s, I chose not to have children for various reasons, the most compelling being the challenges of life. Though I didn't encounter any major issues, the daily struggles and hardships seemed to outweigh the few moments of happiness. I questioned the ethics of bringing someone into the world without their consent. Over time, I realized that my perspective aligns with the antinatalist view point as proposed by Schopenhauer and others. However, sometimes I feel a sense of emptiness due to my decision, as I believe we are biologically programmed to be parents and without that, life can seem even more meaningless. My question to you is what are your views on living a child-free life by choice, Antinatalism, and the existential insignificance that can accompany being child-free?
Well, so I will give you my personal views. Do not try to extend them to other people without their consent, or even take as any implication that I think that other people should have the same views, because these are completely personal views, namely that I am child-free and perfectly happy about that, and I don't make a big deal out of it. I don't think that it is a moral, I don't think there's any moral need or necessity or even vague push towards either having children nor not having children. I don't think morality has anything to do with it. I tend to think that the existence of living creatures, including self-aware creatures like human beings, is overall a good thing. But we have a lot of them, so I don't think there's any need on the part of any individuals to add even more to that number. There's plenty being added all the time. The population of the world is going up right now I think that questions about existential insignificance are not well addressed by either having children or not having children. It's a tough question. I'm not dismissing the question because, as I've said in other contexts, you have to take seriously the fact that even though I exist right now, and once I die, I will no longer exist. And therefore it might be tempting to think that what happens after I die shouldn't matter to me. Right now I carry with me views, projections, predictions, expectations about what might happen in the future, and my current state of mind is very naturally affected by those projections into the future. So I think it's okay to either be proud of the fact that you're raising good children and they might live fulfilling lives themselves after you're gone, or worry about the fact that you're not, or, for that matter, be like I am. Be perfectly happy with the choice that you made not to have children and to do other things instead. You shouldn't be surprised to hear me say there are other ways of getting value and meaning in your life, whether it's right here in the moment or with an eye to the far future after you're gone, that do not rely on procreation. If they do, for many people they will, and that's great. For others, they won't. That is also great. I think that there's many ways to live a meaningful life.
78
2024.0677Freddled Gruntbuggly. I'm tempted. I've sworn off guessing when names are pseudonyms and real. But Gruntbuggly anyway, the question is, I've just finished your excellent book the big picture and have a question about compatibilism and causality. On face value, it seems eminently reasonable that there there are different, compatible ways of talking about the same underlying reality. However, there also seem to be prohibitions about mixing emergent descriptions with more fundamental ones. There are no tables and chairs in the core theory, are there not, however many examples of emergent objects, EG Stars, having effects on microscopic levels, like the fusing of hydrogen nuclei.
Well, two things. One is just for your own vocabulary help going forward. This is not a question about Compatibilism. The word Compatibilism is traditionally used, specifically about free will, about whether or not the idea of free will is in some sense compatible with deterministic underlying laws of physics. So this is more about emergence actually, how do you talk about the relationship between levels of emergent description. There's this question of downward causation within the discussion about emergence. Can higher level phenomena have a causal impact on what is going on at the lower levels? I think that if your lower level is literally particles and fields fundamental physics, then the answer is no. There cannot be separate causal influences of higher level things on lower level things. The example you offer, stars leading to the fusing of hydrogen nuclei, that is not, I think, the right way to think about fusion at the lower level. At the lower level. You can talk perfectly well about the fusion of hydrogen nuclei. You say, if I have these set of hydrogen nuclei and they're moving at these velocities, there will be a certain fraction of them that will fuse together. You never need to refer to the existence of a star. You can also talk at the higher level. You can say here is a star, it has a certain mass, it has a certain composition, it is going to release a certain amount of energy, live for a certain time, et cetera, without ever mentioning hydrogen nuclei fusing. So this is a very clear example to me of two separate descriptions, which are perfectly good in their domains of applicability, which do not need to have any crosstalk between them. But to complicate things, just to open the door a little bit here, if you wanna talk about the emergence of, let's say, society from a "microscopic theory" that was made of people, right? So your microscopic theory is individual humans and your macroscopic theory are collections of humans. Then the distinction, the sort of dividing line between purely macroscopic effects and microscopic ones might not be as clear. It might be possible that in that kind of case, because the microscopic theory is itself non-local and the individual constituents are themselves complex and have many, many internal states and things like that, then it might open the door for a crosstalk between different levels. That's something I'm open to, but I don't actually have my opinions perfectly well pinned down by that, about that question quite yet.
79
2024.0678Hugin says, what do you think are the most likely sorts of experimental observations, if any, that may change our credence about the current ideas of string theory?
Well, I don't know if there are any likely sorts of experimental observations. There are observations we could do that would change our current ideas, our credence about the ideas of string theory. For example, we could discover extra dimensions. You know, if Vafa is right, if there's an extra dimension, a single one that is about microns in size, then that would mean that we would have to, you could have extra dimensions without string theory, but usually in sort of the Kaluza-Klein way of having extra dimensions, they have to be really, really small. Otherwise you would notice them in particle physics experiments, et cetera. So if you discover a large extra dimension, that is very good evidence in favor of string theory, 'cause string theory allows to have these higher dimensional branes, B-R-A-N-E-S, where the fields of the standard model can live. Again, maybe you could have that without string theory, but string theory was certainly responsible for promoting the idea. So that's one way that it could happen. Had we discovered Supersymmetry, that would increase our credence in the string theory, but it would not drive it toward one because you could still have Supersymmetry without string theory. The fact that we have not yet discovered Supersymmetry should lower your credence in the string theory from whatever it was, but how much it should lower, it depends on your likelihood function. So that's up in the air. Personally, I think the thing that we should do with string theory is develop the theory more rather than fret about observations. 'cause we don't understand what the predictions are. We haven't connected string theory to the empirical world yet, that's a job for theorists to try to work harder on.
80
2024.0679Eric Diviji says, I love how your biggest ideas books are so pro math communication. Why do you think people tend to be so afraid of math? Is there something anti-math about our society or educational culture? I don't think it's that.
You know, what do I know? I mean, it could be, but I'm not familiar enough with other societies or cultures to be able to answer this with any data in mind. I think math is hard. You know, math is fun, but it's also hard. Both of those things can be true. Mathematics is not how we were evolved. Our brains are not evolved to do math. Okay? We can do it, it's possible to do it, but it is not something that was useful 10,000 years ago in the same way that is useful now, none of the equations that are in my books were written down 10,000 years ago, and that's a relatively short time on an evolutionary timescale. I was very amused to discover when large language models became so popular that they were bad at arithmetic [laughter], because it's always been interesting that, the human brain is infinitely more, not infinitely, but much, much more powerful as a computer than, a simple pocket calculator or something like that. But pocket calculators are way better at multiplying big numbers than the human brain is. So somehow all of our computational capacity has been optimized for something different than doing arithmetic. And in a computer, it's easy to optimize to do arithmetic relatively perfectly, but somehow large language models also didn't optimize for that. So they also kind of are computers that are bad at math. Now, in that case, it's easy enough to train them to call a subroutine that does math perfectly well, but that's exactly the same as handing you a pocket calculator, right? It's just the extended cognition hypothesis. Anyway, my point is that it's a slightly unnatural way of thinking math, and some people are going to adapt to it relatively quickly. Some people are not. And so I am sympathetic to that. That's completely fine. What I would object to is a psychological attitude that makes it, that brings up some defensiveness about it. And I even have gotten this from a few interviews I've done on podcasts and radio and things like that about my new book, because, people are like, well, I like science, but this is hard. And, they get defensive about it. They blame me like that review of the book that I read at the very beginning of the podcast, right? I think that's a bad attitude that maybe we can blame society for that. If you don't like math then just say, okay, that's fine. There's things I don't like, that's also fine. But you don't need to provide an excuse for it or blame people who do like it or sort of act like it's less important or nerdy or whatever. It's just, it's none your bag. That's okay. Get on with your life.
81
2024.0680Artem Vashartov says, I'm reading your book, Quantum Fields. I have a question about the wave function as a function of momentum, which is the Fourier transform of the wave function as a function of the coordinate. The fact that a snapshot psi of X and T at the particular time moment, T zero contains full information of momentum, looks counterintuitive to me. For example, a bell curve centered at zero is transformed to a bell curve centered at zero. However, to my understanding, the average momentum should depend on how the bell curve psi of X and T evolves in time.
Well, yes it should, but guess what? The Schrödinger equation, I'm guessing Artem, that you're just a, you know a little bit about the math here from the way you asked the question. It's a very good way. So for those of you who are listening who are not listening to the math, apologize for that. But the Schrödinger equation is a first order differential equation in time. That's a fancy way of saying that the right hand side of the Schrödinger equation says d psi dt with a single time derivative. So what that means is that the wave function itself entirely defines, entirely determines how the wave function evolves in time. A wave function, in classical mechanics, in Newtonian mechanics, let's put it that way, Newtonian mechanics F equals ma. That is a second order equation in time, because acceleration is the second derivative of position with respect to time. And that means to solve that equation, you need to give me the position and its first derivative, the velocity. You need to give me two pieces of data to solve a second order differential equation. But a first order differential equation, you just need to give me the wave. The wave itself tells me how the wave evolves with time. Now, so therefore, I could sort of declare victory and say, all the information is in the wave function. I don't separately need to tell you how it evolves in time. Of course, there is something that I should separately tell you, which is there's a real part and an imaginary part, as we briefly discussed earlier in the AMA, wave functions are complex numbers. The phase of the wave function does really matter. And so that might be obscured in here and that's gonna affect the relationship between the position space wave function, and the momentum space wave function. But it is all there if you work it out in detail, it does fit. Don't worry, we're not hiding anything from you.
82
2024.0681Pete Faulkner says, given your trip to France and your obvious love of good food, it seems only natural to ask what is the greatest cuisine in the world, or at least your favorite.
Okay, I am gonna get people upset, but my answer is American. [laughter] Of course, it depends a lot on what kind of cuisine you're talking about in the sense of are we just imagining what people have at home on a typical weeknight? Are we imagining what kind of restaurant you might walk into randomly on the street? Are we imagining fine dining at a place where you would plan to go to and spend a lot of money? Are we talking about the variety of cuisines available, et cetera? There's a lot of different ways you can think about what is the greatest cuisine in the world. You know, I give the United States of America some credit, as much as I give it a hard time, but it's pretty good bringing together, at, cherry picking the best from different areas. When you get to oat cuisine, I know there's a lot of wonderful places in Europe and Asia and all over the world, but I'll put the United States up against any of those places to be perfectly honest, my favorite restaurant in the world is Alinea in Chicago. And I've been to a lot of restaurants. I like going to restaurants. I do think that one of the great things about the US, it doesn't really count as the greatest cuisine in the world question, but many cities will have a whole bunch of really good different cuisines represented, right? Any big city will have all sorts of Asian food and European food, as well as good old American food. I mean, low level, fast food level American food is terrible. It's much worse than you get in Europe. In Europe you can get, maybe not in England [laughter], but in France or Italy, you can certainly get quite good food at quite cheap, affordable everyday places. England, you really have to work, man. I mean, England, you can get amazingly good food, but if you don't pay attention, if you walk into a random restaurant that looks plausible in the UK, you might get in trouble in a way that you probably wouldn't in Italy or France, at least in my experience. And I would put the US in the same boat as the UK in that particular regard. But at the upper levels, the French restaurants are great, but then there's a kind of a similarity there. French cuisine is a thing that is more well-defined, just like Italian cuisine, Mexican cuisine, Japanese cuisine. These are all a little bit less flexible than American cuisine is, like America lets, it helps itself to the best of all these different things. So, and when it comes to great cuisine, that's a benefit. That is a strong point there. So I'm gonna be, yeah, that's, I'll be a little patriotic there against type, but there you go.
83
2024.0682Jay says, to confirm any model of bedrock physics, we must perform experiments, but any experiment is built on bedrock. The very thing we're trying to understand, are we caught, does this mean we're ultimately limited from our ever confirm... From ever confirming physics bedrock?
Yeah, I think that you're not supposed to think about what we do in physics or in science more generally as looking for bedrock. I know it's a nice metaphor, but it's literally a metaphor that I try to debunk in the big picture. You're trying to find ideas that hang together that have some coherence, some consistency between them, whether it's math, physics, data experiment, et cetera. You need a story to tell that includes all these different elements and hangs together, but any one of them could be wrong, right? There's no one bedrock thing that was a dream going back to René Descartes maybe even earlier, but he was the most explicit about it, and he was very careful and realized, no, you can't do it unless you invoke God or something like that. So if you're not gonna do that, bedrock is not what you're looking for.
84
2024.0683Bob Ritchie says, the old advice was to shut up and calculate. Do you spend much time actually doing the math and working the equations or do computers now do most of the grunt work.
I'm still and always have been very much a pencil and paper calculator kind of guy, not a computer calculator kind of guy. You know, the kinds of work that one does affects a lot on what the tools are. If you're doing a numerical simulation, if you're trying to understand galaxy formation, let's say, I've written one paper on galaxy formation, okay, on large scale structure formation, and we did indeed just do pencil paper calculations, [laughter], it was on effective field theory in large scale structure formation. But if you really wanna get the details of what galaxies look like, et cetera, you're gonna need to put it on a computer. So it depends on what you're doing. And in fact, as I am becoming more interested in complex systems and things like that, those are areas where doing simulations becomes more and more helpful. So I'm trying to get good at that. I'm not very good at it right now. I can do the very most basic things, but you know, the time in between when I write a little python script and, the next time I do it is so long that I've forgotten everything about how to do it. It's not yet second nature to me. So I'm still in the process of getting good at that and getting natural at it, whereas just putting pencil to paper or pen to paper and doing the calculations that I'm very comfortable with.
85
2024.0684Jason Hale says, the last 15 minutes of the last podcast with Cumrun Vafa was crazy good. Did you read any papers as you mentioned, or do you have any thoughts on the near future experiments referenced?
I haven't read the paper yet. I linked the paper in the show notes there, the Swamp Landish unification paper, which where he gives a brief overview of these ideas. And then I went on vacation [laughter] So, and then I went on a book tour and then I went on vacation, let's put it that way. So I've not yet had a chance to do that, but I'm looking forward to doing it. It does, look, I think we have to both be interested in this and excited in it if you're that kind of person. And also hold back from being overly excited because, people are always saying, well, I figured it all out, it all fits together. And I have the theory of that makes everything make sense. You have to try to do that. You can't be discouraged, but you can't fall in love too much with any particular one scenario.
86
2024.0685Steve CPW says in your last AMA, you said something like, there are many unanswered questions in theoretical physics. Let's say theoretically that all the questions were answered about gravity, dark matter string theory to name a few. What would be the practical benefit to humankind other than our knowing how the universe works? Would there be any useful application of this knowledge.
Roughly speaking, no. And there might be, you never know for sure, right? You never know that once you have this new understanding, by definition, it's new, you don't have it yet. And so you might realize once you have it that there is some wonderful application of it, but there's no rule that says there has to be an application of it. And, I've made this point before, but fundamental physics has decoupled from technological innovation quite a while ago. I mean, arguably it's been since like the 1950s that discoveries in fundamental physics at the level of, particle physics and gravity and things like that have had any direct relevance to technological progress at all. Muons, pions, things like that, find a little bit of application. But mostly, as I've said, many times, we understand the laws of physics that govern our everyday life regime. And the stuff that happens outside that regime is either very hard to access or goes away, decays away very quickly, et cetera. So you can build a large collider and make Higgs bosons, but you're not gonna make a Higgs boson iPhone because you can't carry around the large Hadron Collider on your wrist, and you need the large Hadron Collider to make the Higgs boson. So it's just of no help. There will be plenty of technological innovations, but they will come from taking the ingredients that we have and putting them together in novel and interesting ways, and that there's an enormous amount of room to do that. So there's plenty of room for technological innovations, but not from, probably not from fundamental physics.
87
2024.0686Henry Jacobs says, I'm banging my head on a QFT textbook, Schwartz, and I'm very lost as to what an observable is in this context. In the case of non-relativistic quantum mechanics of particles, I get it because I could imagine concretely measuring the position momentum spin of a single particle. However, I can't imagine observing an entire field configuration since it's an infinite dimensional object.
Yes, fair enough. [laughter], when we do quantum field theory, as you know, if you're going through a QFT textbook in practice, what we do is we talk about the mathematics of quantum field theory. We use it to then say that you can write Hilbert space as Fock space, which is a collection of many different kinds of particles, superpositions of different numbers of particles, and then we start talking about particles. And that's entirely fine for the most part. You know, there are certain circumstances like inside a strongly interacting proton or neutron or something like that, where you have to do the field theory for its own sake, not just the particles, but mostly you can think about particles. So you don't need to worry about these questions about what are the field observables, and we to simplify our lives, we make this wonderful simplification that you can imagine observing the value of the field at every point in space, all at once. And then you can make a basis for your Hilbert space on the basis from the results of that, the actual field configuration basis. Okay. But of course, as a practical matter, you can't do that. You don't have an infinitely big detector to measure the field all throughout space, all at once. So what you can imagine doing is just measuring the value of the field at a single point. That's something you can do. Phi of X, if Phi is some scaler field or some other kind of field, you could imagine measuring it at some point, and then you get a value. The problem is, as you might if... Once you become familiar with how these things go, if you were gonna get a reasonable answer for the question of what is the field configuration, overall space, asking what it is at a single point is kind of not reasonable because you've sort of infinitely focused in and very, very, very roughly speaking, the uncertainty principle comes in to bite you in the butt. You're asking about what is going on at zero distance, right? Zero extent in space. 'cause you're asking about what happens at a single point. That's gonna mean that, everything else blows up. And so you're typically going to get infinitely big answers. So what people can sometimes do, if you really cared about this question, you could smear out your point in space. So you could invent some function F of X that is zero almost everywhere, but sort of is a bump, like a bell curve or whatever, centered around to the point you care about X. And then instead of measuring phi of x, you can measure the integral of phi of X times F of X. So you basically have a lens or a filter, let's put it that way, that focuses you in on the value of phi at that one point and zeroes out everything far away. It's not clear why you wanna do that. I mean, sometimes you wanna do that because you're trying to literally build the detector or whatever. But if that's true, then probably you can just get away with thinking of everything as particles rather than fields. So there's a interplay that goes back and forth, and you won't find very detailed discussions of any of these issues in most quantum field theory textbooks, because like I said, they just leap right to particles and go from there.
88
2024.0687Johnny O. Says, what does your demon do? Maxwell's Demon violates the second law by Di-Equilibrating, a system without applying work. The Humean demon knows everything that ever happened and ever will ignorant of physical laws, Laplace's demon calculates all that ever happened and ever will by knowing the physical laws in the current state of everything. What does Carroll's demon do?
This is a great question. I love this question. I almost didn't pick it 'cause I don't have any good ideas, [laughter], I don't have, I didn't have, when you asked the question any notion of what my favorite demon would do. But thinking about a little bit and thinking about your examples, which are great, by the way, I actually, there wasn't really a Humean demon, but I know what you mean, that, it makes sense in retrospect that we can talk about the Humean demon. I thought once of like writing a little article or something like that about, 19th century demonology in physics, because clearly that was an idea. Laplace, by the way, never called it a demon. Maxwell called his idea demon. Laplace said, imagine a vast intelligence. I think that Laplace was too much of an atheist [laughter] to invoke demons here, but later commentators called it a demon. So having thought about this question a little bit, here's what my demon would do, in contrast with Laplace's demon. Well, Laplace's demon basically knows all the microscopic information about the world and calculates everything that would happen, and people debate. This literally happened at that Santa Fe Institute workshop I was just at on investigating reality. People debate, does Laplace's Demon know about higher level emergent things? Does Laplace's demon know about temperature or entropy or human beings or intentions or free will or whatever, right? And I think that the simplest answer, there's of course, Laplace's demon doesn't exist. It's fake. So you can make it up, you can answer however you want, but I think the simplest version of Laplace's Demon doesn't know any of those higher level things. It doesn't need to, if you know exactly what all the particles in your box of gas are gonna do, you don't need to talk about temperature or entropy or whatever. You just follow all the particles. So what Carroll's demon does is it thinks about the dynamics of the system, and it tells you not only what the microscopic laws of physics are, but it tells you all of the emergent patterns at higher levels and how they fit together, how you can derive one level from another. So it does what Dan Dennett sort of imagined one could possibly do, figure out the ways to throw away information to coarse grain, to forget some of the things that Laplace's demon knows, and nevertheless have some patterns that tell you what happens in the world. Carroll's demon would be very, very helpful. I wish I could meet Carroll's demon [laughter], and that would be a very illuminating conversation. And a good question there.
89
2024.0688BG 167 says, how does being a celebrity science affect your research and teaching? I imagine there must be some advantages, but also some disadvantages.
I presume this means in the sense that, I have podcasts and books and people will know who I am and things like that. And the answer is, it affects it almost not at all, [laughter] I'm not that much of a celebrity. I've actually, when I've been teaching sometime, I literally would run into a student from my class in the hallway or are waiting for the room to open or whatever, and they said, yeah, I heard you on someone else's podcast yesterday. I keep forgetting like that you're famous outside, like, you're just my professor. I think that's usually what it is. Like I am nowhere near a level of celebrity where most people in the world recognize me on the street or anything like that, or even, let's put it this way, most times that I'm out on the street, no one recognizes me. The vast, vast, vast, vast majority. There are occasional encounter examples, which are pretty hilarious, but I'm pretty safe from true celebrity issues. So the students in my class often have never heard of me before. The fellow scientists, of course, they have heard of me because I'm a fellow scientist, but they might have also known that I have books and things like that. But the thing is that, how you think about a person depends on the aspects of their life that you are most familiar with. So if someone, I have a lot of younger scientists who, physicists who know me because of my general relativity textbook, right? That was their first exposure to me. So they think of me as the textbook guy, the general relativity textbook writer. People my age think of me from my research, right? That's how they got to know me, medic conferences, things like that, et cetera. So when I'm talking to scientists, they for the most part did not get to know me first through podcasts or books or whatever. So, in their minds, yes, I also write books and have podcasts, et cetera. But mostly I'm a scientist writing science papers, right? And whereas people who are fans of the podcast or fans of the books barely know about the physics papers that I write, or the philosophy papers for that matter. So who you are depends on the lens that people see you through.
90
2024.0689Georgio says, given that we have trouble understanding how complex life can arise from simple physical and chemical processes, is it respectable to have a high credence in the theory that the earth might have received some sort of interstellar pollination from an alien civilization sending their seeds out.
You know, you're allowed to have whatever credence you want, and that it's very speculative. I don't think that, I mean, it's respectable to have a high credence. I don't think you should. I don't have a high credence in that. Basically because it, given that we have trouble understanding how complex life can arise from physical, simple physical and chemical processes, that's just as true on alien worlds as it is on our world, right? You haven't made anything easier by saying that that unknown process happened somewhere else. You've made it harder because you say, well, it happened somewhere else, then it had to travel across interstellar space, which sounds difficult. You know, again, it's very possible, like maybe it only happens once in the universe, and then this alien civilization spreads its seeds out. But that, I mean, that sounds a little weird, why would the alien civilization do that? Why would they spread out little molecules or primitive organisms in a way that they wouldn't even know what the results of that were? Like, why not just send spaceships or something like that? I think far in away, the most likely thing is that life began here on Earth. We'll have to find out whether or not that's true or not. Well, it's still a research problem there.
91
2024.0690Schlier says, if you were given the power to intervene in human history and cause humans to never be able to access fossil fuels, would you do so?
No. I would certainly not do so. Fossil fuels have been enormously helpful in building technology and growth of productivity and wealth, and therefore other kinds of technological innovations like medicines and things that to make our lives generally better. Now, we have clearly overused them. Yeah, that's true. If I had the power to intervene in human history, I would've started the transition to cleaner energy much earlier, let's put it that way. But it would've been hard, it took a while to get to the technological point where renewable energies are competitive with fossil fuels in the way that they are now. But again, if I could magically intervene in human history, that is what I would cause to start happening much earlier.
92
2024.0691Johnny says, do you listen to music while you work or full silence? Curious what music you use to focus, if any.
I usually just work in silence. I get distracted, so let's put it this way. I can't really do work if there's music going on that has lyrics, [laughter], if the music has lyrics, then I'll start thinking about the lyrics and your best, whether it's doing writing or doing physics, or doing philosophy or whatever. It's for me personally, if I'm going to make progress, I need to be somewhat very carefully focused on that particular thing, and lyrics get in my way. They engage the part of my brain that should be doing work and that's counterproductive. Sometimes I will listen to instrumental music. I have a playlist on iTunes that of, writing music, but most of the time I forget to turn it on. But sometimes I do. You know, it's a weird mix of, some classical things, some instrumental jazz things, but not like too wild stuff and some, world music kind of things again, but all super instrumental background that it is, it generates a good feeling in the background without distracting you from the important work you're supposed to be doing. I know that other people are completely different. Other people like to blare really loud music with lyrics, and that helps them work, but it depends very much on the kind of work you're going to try to do.
93
2024.0692Nanu says, I truly enjoyed listening to your episode with Dr. Vafa. It was interesting listening to him defend the potential existence of extra curled up dimensions in the real world. As a theoretical physicist over the decades, how did your intuition shift and mold after big discoveries, like the acceleration of the universe and others? In other words, how do you approach new ideas with minimal preconceived notions based on senses of your body? Does it take practice in time like a muscle?
I think in the case of, I mean, it's an interesting question because the kinds of big discoveries that have happened over my scientific career have not been that difficult to incorporate into our intuition. The acceleration of the universe was the biggest, but I had literally already written a review article on the possibility of that happening. So it wasn't like I had to radically changed my preexisting knowledge base. Something like the invention of quantum mechanics or the big bang or relativity, that would've been a much bigger shift of perspective. And I cannot honestly say how I would have reacted to that. You know, as I often have said in some very real sense, physicists still haven't taken quantum mechanics to heart, and I would like them to do that. So there's absolutely some barrier to completely shifting your intuition. Quantum mechanics is the clearest example, because there's this difference between the machinery we use to actually do calculations and make predictions, and then the observations that we actually see at the end of the day. And I think people really struggle with distinguishing between those things and figuring out how they could fit together well. But other people disagree. [laughter]
94
2024.0693Nicholas Delan says, what is the most underrated or overrated thing about Paris?
I don't know. You know, I think in practice, like I said earlier, I think, if you're there at a heavy part of tourist season, going to museums is a little bit overrated. There's a lot of waiting in lines. Like we foolishly had the idea. We were walking around in the morning and we're like, okay, oh, the Louvre is right there. Let's just go to the Louvre and check it out. And it's been years, and the line was like three hours. And, we're not standing in line people at this stage of our lives. So we did not do that. And even at the, I don't know, the Algérie, which is this tiny little museum, it was like an hour long line. Why bother with that? You know, I like Paris because you can walk around, you can look at stores and shops. You can sit at the cafe, you can have a good meal, you can have a glass of wine. You can enjoy the river and everything. You can enjoy the neighborhoods. One of the underrated things about Paris is, I don't know whether this is just because it's a big enough city or whether it's European or whether there's a lot of history or whatever, but you can get free entertainment very easily. Some of my favorite experiences in Europe have been going to concerts in cathedrals that are, held by candlelight or whatever, just put on for free. They're not the highest level of classical music, but they're very good and the atmosphere is unbeatable, right? And you can just walk in on the spur of the moment and enjoy something like that. So I think that, but as I've said before, and it's not very helpful to say, everyone's version of what to do in a city like Paris is gonna be different. So try different things. Like I've never gone to a jazz club in Paris. I think that would be fun, but I just, I've never had the chance to do that. So, someday I'll go back.
95
2024.0694Mark Slight says, I read that nowadays your applied physics guys have gotten interference patterns out of quite large molecules through the slits. This struck me as odd, but I guess that's just bad intuition. Is this unsurprising? What are the theoretical and practical limits or difficulties? Could I in principle, interfere with myself?
Yeah, you could in principle interfere with yourself. You have a wave function. You diffract a little bit when you walk through a doorway every time. But that little bit is a very, very, very little bit to actually, the obstacle to you interfering with yourself in a double slit experiment is not just that your wave function, because you are very, very massive. The, it's almost impossible to get your wave function to pass through two slits. Okay? That's one problem. Your wave function is far, far, far too localized. Not to mention that your body is bigger than the slits, so you need to make the slits as big as doors. So you're not gonna get a lot of interference. But in principle, it's there. There's another problem, though. It's not just that your wave function is very localized 'cause you're massive. It's that you're made of a huge number of particles, right? And all these particles are constantly vibrating and giving off radiation and interacting with the rest of the world. So basically you would decohere here right away. It would be the version of the double slit experiment where you're observing all the time, which slits you go through because you're interacting with the rest of the world and becoming entangled with it. And then finally, even if you got rid of that effect, the fact that you are evolving over time in sort of different sort of jiggly ways means that the you that hits a detector on the other side of the slits would not be the same. The one that goes through the left slit and the one that goes through the right slit would've maybe very, very likely evolved in slightly different ways. So when you have a single particle going through two slits, there's no internal degrees of freedom for a single particle, or there's not that many of them. So it's crucially important that in order to get interference on the other side of the double slit experiment, you have contributions to exactly the same final state of the particles, contributions to the particle being in an exactly specified quantum state at this point on the detecting device, having gone through one slit or the other. But if you go through one slit or the other, chances are you'll evolve in slightly different ways going through the left slit and going through the right slit. So you will not end up at the detector screen in exactly the same quantum state, and therefore you will not interfere. So that's why your intuition is not very good at this, because it is very, very, very far away from being practically reasonable.
96
2024.0695Sandro Stuckey says in quantum fields, you explain how a quantum field consisting of a single mode has discrete energy levels that look like those of a quantum harmonic oscillator. You also say that any field configuration can be obtained as a sum of possibly infinitely many modes. But it's not obvious to me that the solution of the Schrödinger equation or the resulting energy levels for the sum of two warm modes still looks like those for a single mode. There seem to be non-linear terms in the energy density. How do we still get a harmonic oscillator? Or sorry, do we still get harmonic oscillator? If not, does the nice picture of discrete energy levels correspond to discrete numbers of particles break down once we have particles corresponding to different modes?
This is actually a very, very subtle question. If you look carefully at the discussion in quantum fields, I'm talking about a free field theory very, very specifically, which means I have a very, very specific energy of the theory, which is just the kinetic energy for, let's call it phi, some scalar field, plus the gradient energy for as phi changes from place to place, plus a very specific kind of potential energy, namely phi squared times some parameter. And that means that all of that terms, all the terms of that energy density are quadratic in phi. They're order phi squared. That means that the equations of motion that you get by roughly speaking, differentiating with respect to phi are linear in phi. That means there are no non-linearities. You can solve the equation exactly. And that's when you get this nice behavior of particles. In the real world where you have interacting fields. So you have terms in the energy density or Lagrangian that are cubic or quadra or Quartic or even more whatever. Then as you point out or as you're gesturing toward, you will not have this ability to cleanly solve the equations and get energy levels like a harmonic oscillator. You will have non-linearities, and they will be important. So usually you say, okay, but those interactions are weak, they're small, and that's when we can do perturbation theory. Now you're halfway on your way to inventing Feynman diagrams. That's exactly what Feynman diagrams do. They answer the question, how do you extend from a completely free field theory point of view where you can identify the particles precisely to an interacting field theory point of view, where the particles are gonna bump into each other and potentially change their identities. This turns out to be super subtle because the particles, the fields rather, are constantly interacting with each other. And so you have to invent an elaborate formal structure of scattering theory where you start, you literally set up your initial conditions by imagining that you have turned off the interactions, you take, so there's some coupling constant, like a fine structure constant of electromagnetism. You set it equal to zero. And so you do have a free theory and you can sensibly say, oh, there's a photon, there's an electron or whatever. And then you send them toward each other. And then there you invent a mathematical technique by which you can gradually turn on the interaction before the particles get close to each other, and then they interact and then they leave, and then you turn off the interaction again. And then you measure at the end of the day. It's slightly non mathematically respectable, this whole thing. There's a lot of work that goes into making it even plausible. And as a practical matter, it works, right? We get the right answers for doing this. But taking that all of those complications seriously is in fact very, very important. We didn't have time to go into it in quantum fields, but a real quantum field theory book pick up Peskin and Schroeder, it will actually go about this in great detail. The LSZ reduction formula, the interaction picture, words like that will appear in the discussion. Okay, we've gone a long time here.
97
2024.0696The final question comes from Dan Berliner. What is the secret to making a good pizza crust?
Very good question. I feel like the underlying theme of today's [laughter] AMA is lots of different things are possible. Lots of different ways to be a good person are possible, lots of different scientific ways of operating are possible. It's also possible, lots of different ways to make a good pizza crust. And I enjoy all sorts of different pizzas. I know that, it's popular to pick a certain kind of pizza and dogmatically insist that it is the only right one, whether it's, Brooklyn-style pizza or Italian-style pizza, or Chicago-style pizza or whatever. I am not that guy. My love of pizza is ecumenical. I love all these kinds of pizzas. It's great to like go to Italy or go to some tiny little shop. And I visited recently the University of Pittsburgh, and I gave talks there, et cetera. I was taken to a nice little pizza shop where, the owners, the proprietors are clearly right from Italy, and they make you, that personal style thin crust pizza with really fresh ingredients, which is amazing. You know, I love that. It is what you get in Italy. But also I spent seven years living in Chicago eating deep dish pizza, which as cartoonist Reuben Bolling once put it, is some kind of bread based lasagna food [laughter] It's not really pizza at all. But I ate an embarrassingly large amount of deep dish pizza when I was in Chicago. And when I make pizza myself, I've actually not perfected like the thin crust Italian style. I had a friend back in LA who did, who like owned a pizza oven and he would have parties and we would make our individual pizzas. And that was kind of amazing. But I have not been able, I have not tried too hard, but that's actually one of the, one ambition that I have. I do have a way of making pretty killer pizza, which is very, very simple. You can go to Whole Foods and they sell you like a little plastic baggy of pizza dough. So you don't even have to make the pizza dough yourself. And you might fear that this pizza dough is not very good, but in fact it's quite good. And they will also sell you the sauce. You can do better than the whole foods sauce. You can get RAO's R-A-O's, RAO's, pizza sauce, some mozzarella cheese, pepperoni, onions, whatever you wanna put on it. And all you have to do is get a good cast iron skillet, coat the cast iron skillet with some olive oil and maybe some garlic and some breadcrumbs if you have those. And then smush down the pizza dough into your cast iron skillet. Cover it with your cheese and your toppings. Put the whole thing skillet and pizza and everything in the oven at 400 and some degrees, until it's done. I don't know, it takes 15, 20 minutes to do it. So it's deep. It's not Chicago-Style deep dish pizza because that's like layers of things, right? This is pan pizza, but it is so good. [laughter] This cast iron skillet pizza with Whole Foods Pizza dough is amazingly good and super simple. So that's my level of commitment to making good pizza at home. I would like to be better at making sort of flatbread Italian style pizza, but I'm young. God is not finished with me yet. I have not yet acquired all the skills I will acquire before I die. So life is a journey. I'm looking forward to living it.
98
2024.051Alex Tough says, "Can you comment on the college campus protests currently ongoing? Where do you draw the line between freedom of expression and illegal activity? If you were a university leader or administrator, how would you handle the situation differently, if at all?"
This is obviously a complicated situation. People don't like complicated situations. They want to put situations into boxes that are relatively black or white. I'm all in favor of campus protests. I think this is one of the things that college students are good at doing. Not putting up with some established way of being and trying to make the world a better place and getting indignant about it. I mean, going to college if you do it right, it can be quite an experience intellectually, emotionally, in terms of how you think about the world. For many people, when you go to college, you're living there, this is the first time in your life or maybe one of the small numbers of times in your life where you're in a completely different environment. Completely different sets of people. You're not with the same people over and over again. You're maybe not living with your family and you're exposed to all these new ideas from other students, from professors and classes, from books and things that you're reading and so forth. It is no surprise that college students who are also at the same time, still young and energetic, right? They're not settled into ruts, are going to be outraged by things happening in the world that they are catching onto and realizing are unjust and they want to change. So in general, I think that the first response of college administrators to protesting students should be that that's good. That's part of what the college experience is all about. Of course, another feature of being a college student is that you're not that wise in the ways of the world as yet, right? You can get overly enthusiastic about things because you've recently discovered them and you're a new convert. And that happens. There was a thing going around on the internet, which purported to be a list of demands from students at Cornell as part of the recent protests. I don't even know if it was legit or not, but it is emblematic of something that does happen, which is that despite the fact that the protests were nominally about the conflict between Israel and Palestine right now, there was a bunch of demands stuck in there about indigenous lands and having students have the right to help choose faculty members and a bunch of things that had nothing to do with what was going on in Gaza, which illustrates, I think, a broader problem, which is that these kinds of protests are often not very well targeted to actually make a difference. To actually get something done. It is often unclear what the goals are, what the realistic things that could happen would be. And often there's a spirit that compromise is bad, right? There's a weird tension, there are students protesting on the basis that they would like to see Israel and Palestine sit down and work out their differences in compromise but they themselves are unwilling to compromise with Joe Biden, right? 'Cause he's insufficiently radicalized about these particular ideas. This is... I'm getting onto my own little hobbyhorses here. I think that in the modern world, we're not very good at compromising. We've given up or somewhat lost that ideal of democratic governance where we're supposed to talk to our enemies, right? You don't make peace with your friends, you make peace with your enemies. And that's something that student protesters are not especially good at. The strategic part of politics, who to compromise with, how to get your goals actually to come true in some way. So I think that there's... You have to understand that the protests are going to be kind of messy and chaotic and and etcetera. And that's also okay. If the protests are just sitting in the yard singing some songs, giving some demands out there, then I think that the university administrators should bend over backwards to let them happen. One very obvious thing is that a lot of administrators just don't talk to the protesters. Their first instinct seems to be to call in the cops, which is not good for anybody. In Philadelphia, I know when University of Pennsylvania called in the cops on the protesters, the cops said no. They're like, "We don't see any evidence that anything actually violent or dangerous is happening here. Just go and talk to them." And you know that when the cops are saying like, "You're being a little bit too violent in your reaction here." Then maybe you should rethink your strategy a little bit. Of course, it is possible for protesters to go overboard. Of course... Well, then there's many different ways of going overboard. One way of going overboard is just to say horrible things, right? There have been multiple examples of protesters saying things online or on videos that are terribly antisemitic or racist or whatever. Those people exist. Those people are real, right? And they're saying terrible things. And that doesn't necessarily undermine the legitimacy of the protest overall or of the cause that all sides have bad people saying bad things about them. You should think about the best arguments for both sides, not the worst arguments. When it comes to the students like damaging property or stopping classes from going on, then it becomes a legitimately difficult case-by-case basis kind of call, I think. On the one hand, it is true that protests in order to get their message out, will sometimes break the rules, right? That has been true forever. Certainly the Boston Tea Party did not play by the rules that had been laid down. The various civil rights demonstrations did not always play by the rules that the local communities had put up. Sometimes to make progress, you have to break the rules. Occupy a building, stay out even though you're told to go home, things like that. On the other hand, again, I would personally want the protesters to think strategically when they are getting in the way of people, when they're stopping people from having classes, for example or whatever, are the people who they're inconveniencing actually the ones who have a power to do something, right? If you're going on strike in a more general circumstance, are the people who that strike is inconveniencing, the actual ones who have the power to offer you better working conditions or salaries or whatever. I think that these strategic, "Can you actually make the world a better place?" questions should be in the forefront of protesters. Thoughts about how to best organize and what strategies to take. I am someone who doesn't get that interested in sort of the virtue signaling aspects of doing a protest. I do think that a lot of protesters are just there to sort of have their voices be heard more so than to make the world a better place. I personally I'm about making the world a better place. So that's the balance that you have to strike. And I don't think I... I'm not sure that there is a simple set of rules you could hand down in a sort of bureaucratic way that would cover all of the cases. You have to think on a case-by-case basis, what is actually going on. But the overall thing is I support the protesters. I support their rights to have their voices be heard. I'm pretty much on one extreme when it comes to letting voices be heard, if that's what you're into. I think that you should let Nazis give talks on college campuses if they're invited by some legitimate college campus group. I think you should let people with all sorts of terrible ideas give talks and let them talk, and then ignore them if you want to. Give another talk, if you want to do that and then let people think about it. That's what I would think is the ideal situation. We can't always achieve it, but we can try to work toward it. That would be my goal.
99
2024.052Mickle Pickle says, "Your recent guests have pointed to counterfactual reasoning as a key element of humid advancement, either as mental time travel or considering the consequences of different initial conditions or empathy for the plight of others, etcetera. I've begun to notice how often I do it and to suspect it's true that educability, for example, and perhaps our overall success as a species depend upon it. However, I can't tell if it is required or most efficient or if it just happens to be what I personally do. How important do you think counterfactual reasoning is to effective prediction as individuals or to our past and future advancement as a species?"
It's hard to answer. I mean, the answer is super important, basically. But when you ask, "How important is it?" I'm not quite sure how to quantify it, right? But I do think that this idea of counterfactual reasoning, I think maybe the first time we talked about it was with Malcolm MacIver when he was talking about the initial evolutionary development of the capacity for counterfactual reasoning when fish climbed onto land and could suddenly see a lot further than they could before. And yeah. You're right. We've talked about people who have been thinking more specifically about how humans use those capacities. It's hard to imagine something... Oh, Karl Friston, we actually also talked with about that issue 'cause I was... I remember joking with him that of my two cats, Ariel and Caliban, one seems to be capable of counterfactual reasoning and one does not. Caliban just seeks the local minimum. He just wants to be as happy as possible in the moment. I don't think the idea of other moments ever occurs to him. Whereas Ariel is the stereotypical cat who when you open a door that she demands to have open, she's like, "I don't know. Do I want to want to open that? Do I want to want to walk to that threshold? I'm not quite sure." She's always thinking about the possible bad consequences, etcetera. And our feral cat, our outdoor cat, Puck, who is a hung around outside our yard for a while, he's super cautious, right? 'Cause this is what is keeping him alive. So he doesn't do anything until he thinks about it 20 different ways. So somewhere that's close to when this capacity, in some minor key, was first developed evolutionarily. But anyway, yeah. Absolutely possible, absolutely crucial, because I think that I suspect... And I'm just making stuff up. I'm not an expert on this. But I suspect that the parts of the brain that are good at in general, abstract symbolic reasoning, are either the same as or closely related to the parts of the brain that are good at counterfactual reasoning. I don't know if we've ever done a podcast on it, but back years ago, I organized workshops on the Arrow of Time where we were taught that you can do FMRI studies in the brain and you can show that when people think about possible future events, the literal part of the brain that is doing that work, conjuring up an imaginary hypothetical scenario is the same part of the brain that sort of puts a scenario together when you're remembering things that actually did happen. So there's a sort of separate modules in the brain for storing information about what happened in the past versus literally putting on the show. When we store the data of past memories, we don't store a videotape. We store something closer to a screenplay and we put on a little show in our brains. And so that gives us a the capacity to put on a hypothetical counterfactual show. And I would not be at all surprised if that was a crucial step towards developing abstract symbolic reasoning, which is of course, super duper important for everything that humans are able to do. It gives us that sort of touring machine universality that lets us think about the universe in perhaps at least a maximally powerful way. Again, I don't know the details, you should ask someone who knows about this stuff.
100
2024.053Colleen Edwards asks, "I asked this question on your quantum field theory thread regarding your upcoming book, which I know you still have to get to, but I just can't stand the anticipation. So I'll ask here in hopes of having an answer sooner. So what Colleen is referring to is that next week's podcast will be a solo podcast based on ideas from my upcoming book, Quanta and Fields Volume II of the Biggest Ideas in the Universe. The idea of the book is to explain quantum mechanics and more importantly, quantum field theory in particle physics in a way that is accessible to everyone but that shows you the equations, right? That's the trick. And we'll see whether the trick worked, but that's the goal in a short, powerful package. But there's so many ideas in quantum field theories that at the heart of modern physics, that even in a solo podcast or a talk that I go around in giving and give, I cannot talk about all the good ideas. So for Patreon supporters, I ask the questions like, "What are the most important ideas you would like me to talk about?" That is what Colleen is referring to when about the quantum field theory thread. Just one of the many secret benefits of being a Patreon member here at Mindscape. Oh, I forgot to say. I got distracted by other issues. But of course, these AMA questions are supported by Patreon members, and they're the ones who get to ask them. If you would like to be a Patreon member, go to patreon.com/seanmcarroll and you can see all these secret threads and ask the questions also, as well as get advertising-free versions of the podcast. So Colleen goes on. "As a non-physicist, I'd like to know why does the field vibration change to a particle when it is observed? This concept is still tough for me to grasp. How can vibration change to a particle just like that? Is it possible to observe the vibration as it changes into a particle?"
So I think I will talk about this, hopefully. I do talk about it in the book in great detail and I'll hopefully talk about it next week. But I think the rough thing to think is that it's not quite, that a vibration is changing into a particle. The thing about quantum mechanics is that what you observe is not what is, right? If you think that what is, is the wave function of whatever things you're thinking about, what you see when you look at the wave function depends on certain observables. You see position, you see velocity. So forget about quantum fields. Even for an electron. If I have the wave function of electron, it could be spread out all over the place and it's a particle, but it's a quantum particle, so it has no position, but I can observe the position and I get an answer. I see, "Oh, there is a particle with a certain position," or I can observe the momentum and see, "Ah, there is a particle with a certain momentum." Can't do both at once because of the Heisenberg Uncertainty Principle. So the real question is, why when we observe quantum fields, do we see particles? Why is it that the thing we observe looks particle-like? And so it's not that it transforms into it. I mean, it kind of does because the wave functions collapse, and that's how we talk about it. But it's not like it's doing it by itself. It's a function of that interaction. It's how you interact with the quantum fields that is making you see them as particles. And there's a longer story there and involves simple harmonic oscillators and things like that. But I'll just put a little tiny idea into your head to think about that will soften the journey to get there, which is think about how electrons... Particles. Forget about the quantum field things. Just think about how electrons have energy levels in atoms, right? The various orbitals that you study, if you're a chemist. There's a lowest energy level. There's the higher energy levels that are sort of more funky looking orbitals and so forth that gives rise to the periodic table and all that stuff. Where do those come from and what is the role of these orbitals? You might say, "Well, look, I have a wave, the wave function of the particle, and it's surrounding the nucleus of the atom." And I can imagine all sorts of profiles for the wave, right? All sorts of wavy kinds of things. But when you have the equation in front of you, the Schrödinger equation, what you find is that only certain profiles for that wave function have definite energies, right? There's a lowest energy answer to the question, "What is the solution to the Schrödinger equation?" And there's a discreet set of higher energy states. There's a discreet set of orbitals. So even though in principle the electron wave function can have any shape at all, in practice, dynamically, it goes to lower and lower energy states and those energy states have definite shapes, okay? And the definite shapes they can have form a discreet set, even though the underlying wave is continuous. Much like, and you will hear this analogy again, much like plucking a violin string with its ends held down. There is a fundamental tone and there are harmonics that have more vibrations in them, but they come in a discreet set. So the softening the blow kind of hint I will give you is quantum fields are also like that. Even though the quantum field need not be isolated around a nucleus of an atom or anything like that, it has vibrations that have definite energies, and there's a discreet set of them. And it turns out, this is a... I literally say in the book, it's a walks like a duck quacks like a duck situation. The discreet energy levels that a quantum field can have turn out to have all the properties of particles, right? There's a state with zero energy or the minimum energy state, which we interpret as empty space, there's a state with one quantum worth of energy, and you can sort of look at it from different reference frames and it looks like it has momentum and it looks like a one-particle state. And then there's the next highest energy states that have at least twice as much energy. Why? Because they're interpreted as two-particle states, etcetera. So it's a combination of those two things. Number one, solving the Schrödinger equation gets you these discreet energy levels that kind of have particle like behavior. And then number two, when you measure, when you observe the quantum field, you're generally looking for things that have some discreet amount of energy. Some definite amount of energy, I should say. The wave function itself of the field might be a superposition of various different states of energy just like a radioactive nucleus can be in a superposition of, "I've decayed and I've not decayed." But both of those are going to be things you observe. When you observe, you look for locations of things and the locations, the sort of localized excitation map onto these discrete energy levels, and we interpret them as particles. So it's really that they always are waves. They look to us as particles because of the dynamics of the Schrödinger equation applied to the quantum fields. Hopefully that's at least, a little bit of help.